Решите уравнение x 8 5: Решите уравнение -x=8/5 (минус х равно 8 делить на 5)

Содержание

Открытая Математика. Алгебра. Уравнения, содержащие модуль

Самый распространённый, а иногда и единственно возможный метод решения уравнений с модулем – раскрытие модуля согласно определению:


|x|={x,-x,x≥0,x≤0.

Решите уравнение |x – 5| – |2x + 8| = –12.


Выражения, стоящие под знаком абсолютной величины, обращаются в нуль при x = –4 и x = 5. Значит, нужно рассмотреть 3 случая:


1) x ≤ –4; 2) –4 < x ≤ 5; 3) x > 5.


Получим три уравнения, в каждом из которых на неизвестное наложено ограничение. На рисунке схематично показано, какой знак будут иметь подмодульные выражения на каждом из трёх промежутков.




  1. x ≤ –4. В этом случае 2x + 8 < 0, x – 5 < 0. Следовательно, {|2x+8|=-2x-8,|x-5|=x+5.
    С учётом этого уравнение принимает вид




    -2x-8-5+x=12⇔x=-25.

    x = –25 удовлетворяет ограничению x ≤ –4.



  2. –4 < x ≤ 5. 2x+8-5+x=12⇔3x=9⇔x=3.
    Этот корень удовлетворяет нужным ограничениям.
  3. 3. x > 5. 2x+8-x+5=12⇔x=-1.
    Этот корень не удовлетворяет нужным ограничениям.

Ответ. −25; 3.


Этот метод удобно применять, когда подмодульные выражения довольно просты (линейны), и можно сразу понять, где они обращаются в нуль. Рассмотрим простейшее уравнение с модулем вида

|f (x)| = g (x),


где функция f (x) проще функции g (x). Это уравнение равносильно следующей системе уравнений:

|f (x)|=g (x)⇔[{f (x)≥0,f (x)=g (x);{f (x)<0,f (x)=-g (x).

Убедиться в справедливости этого утверждения можно, перебрав все возможные варианты.

Если же под модулем стоит функция, найти корни которой затруднительно, то условие равносильности можно переписать так:

|f (x)|=g (x)⇔{g (x)≥0,[f (x)=g (x),f (x)=-g (x).

Решите уравнение 2|x2 + 2x – 5| = x – 1.


Этому уравнению соответствуют два уравнения 2(x2 + 2x – 5) = x – 1 и 2(x2 + 2x – 5) = 1 – x, среди корней которых нужно отобрать удовлетворяющие условию x ≥ 1. Имеем:


1. 2×2+3x-9=0.
Корни этого уравнения x=32
и x = –3, из которых подходит первый корень.


2. 2×2+5x-11=0.
Корни этого уравнения x1, 2=-5±1134.
Опять подходит только первый корень, так как второй заведомо отрицателен.



Ответ. x=32,  x=-5+1134.


В случае вложенных знаков модуля применим этот метод несколько раз. Здесь тоже можно рассмотреть весь набор получающихся при раскрытии модуля уравнений среди решений которых содержатся решения исходного уравнения, а потом отобрать из всех полученных решений подходящие хотя бы с помощью проверки.

Решите уравнение ||x3-x+1|-3|=x3+x+1-7.


Все корни исходного уравнения содержатся среди корней двух уравнений

[|x3-x+1|-3=x3+x+1-7,|x3-x+1|-3=-x3-x+1+7,

которые можно переписать в виде


[|x3-x+1|=x3+x+1-4,|x3-x+1|=-x3-x+1+10.



Аналогично, каждое из этих уравнений распадается на два:

[[x3-x+1=x3+x+1-4,x3-x+1=-x3-x+1+10,[x3-x+1=-x3-x+1+4,x3-x+1=x3+x+1-10,

что приводит к четырём уравнениям:

[x+1=2,x3=5,x3=2,x+1=5.



Отсюда получаем 4 решения: x1=3,
 x3=53,
 x2=23,
 x4=24,
 среди которых содержатся корни исходного уравнения. 1-й корень, очевидно, удовлетворяет уравнению. Это проверяется легко. 2-й и 3-й не походят, так как правая часть исходного уравнения при этих значениях отрицательна. 4-й корень тоже является лишним, так как этот корень должен удовлетворять уравнению (*), а при этом значении его правая часть отрицательна.



Ответ. 3.


Пример №90 из задания 13 (профильный уровень) ЕГЭ 11 класс

а) Решите уравнение `(x+3)^2/5+20/(x+3)^2=8((x+3)/5-2/(x+3))+1`. 2+3x-10=0`;

`D=9-4*(-10)=49`;

`x=(-3+7)/2=2`;

`x=(-3-7)/2=-5`.

б) Отберем корни, принадлежащие отрезку `[-6; -4]`.

Сразу видно, что `2` не входит в данный отрезок, а `-5` входит.

`(-1-sqrt(65))/2=` `-sqrt(1/4)-sqrt(65/4)=` `-sqrt(66/4)=` `-sqrt(16,5)`;

`(-1+sqrt(65))/2=` `-sqrt(1/4)+sqrt(65/4)=` `sqrt(64/4)=` `sqrt(16)`;

`-6=-sqrt(36)`;

`-4=-sqrt(16)`.

Теперь видно, что `-6

Получились следующие корни: `-5; (-1-sqrt(65))/2`.

Решение №2 (скан):

$IMAGE2$


Ответ: а) `(-1+-sqrt(65))/2; -5; 2`;
б) `-5; (-1-sqrt(65))/2`.


Общие сведения об уравнениях

Уравнения — одна из сложных тем для усвоения, но при этом они являются достаточно мощным инструментом для решения большинства задач.

С помощью уравнений описываются различные процессы, протекающие в природе. Уравнения широко применяются в других науках: в экономике, физике, биологии и химии.

В данном уроке мы попробуем понять суть простейших уравнений, научимся выражать неизвестные и решим несколько уравнений. По мере усвоения новых материалов, уравнения будут усложняться, поэтому понять основы очень важно.

Предварительные навыки

Что такое уравнение?

Уравнение — это равенство, содержащее в себе переменную, значение которой требуется найти. Это значение должно быть таким, чтобы при его подстановке в исходное уравнение получалось верное числовое равенство.

Например выражение 3 + 2 = 5 является равенством. При вычислении левой части получается верное числовое равенство 5 = 5.

А вот равенство 3 + x = 5 является уравнением, поскольку содержит в себе переменную x, значение которой можно найти. Значение должно быть таким, чтобы при подстановке этого значения в исходное уравнение, получилось верное числовое равенство.

Другими словами, мы должны найти такое значение, при котором знак равенства оправдал бы свое местоположение — левая часть должна быть равна правой части.

Уравнение 3 + x = 5 является элементарным. Значение переменной x равно числу 2. При любом другом значении равенство соблюдáться не будет

Говорят, что число 2 является корнем или решением уравнения 3 + x = 5

Корень или решение уравнения — это значение переменной, при котором уравнение обращается в верное числовое равенство.

Корней может быть несколько или не быть совсем. Решить уравнение означает найти его корни или доказать, что корней нет.

Переменную, входящую в уравнение, иначе называют неизвестным. Вы вправе называть как вам удобнее. Это синонимы.

Примечание. Словосочетание «решить уравнение» говорит самó за себя. Решить уравнение означает «уравнять» равенство — сделать его сбалансированным, чтобы левая часть равнялась правой части.


Выразить одно через другое

Изучение уравнений по традиции начинается с того, чтобы научиться выражать одно число, входящее в равенство, через ряд других. Давайте не будем нарушать эту традицию и поступим также.

Рассмотрим следующее выражение:

8 + 2

Данное выражение является суммой чисел 8 и 2. Значение данного выражения равно 10

8 + 2 = 10

Получили равенство. Теперь можно выразить любое число из этого равенства через другие числа, входящие в это же равенство. К примеру, выразим число 2.

Чтобы выразить число 2, нужно задать вопрос: «что нужно сделать с числами 10 и 8, чтобы получить число 2». Понятно, что для получения числа 2, нужно из числа 10 вычесть число 8.

Так и делаем. Записываем число 2 и через знак равенства говорим, что для получения этого числа 2 мы из числа 10 вычли число 8:

2 = 10 − 8

Мы выразили число 2 из равенства 8 + 2 = 10. Как видно из примера, ничего сложного в этом нет.

При решении уравнений, в частности при выражении одного числа через другие, знак равенства удобно заменять на слово «есть». Делать это нужно мысленно, а не в самом выражении.

Так, выражая число 2 из равенства 8 + 2 = 10 мы получили равенство 2 = 10 − 8. Данное равенство можно прочесть так:

2 есть 10 − 8

То есть знак = заменен на слово «есть». Более того, равенство 2 = 10 − 8 можно перевести с математического языка на полноценный человеческий язык. Тогда его можно будет прочитать следующим образом:

Число 2 есть разность числа 10 и числа 8

или

Число 2 есть разница между числом 10 и числом 8.

Но мы ограничимся лишь заменой знака равенства на слово «есть», и то будем делать это не всегда. Элементарные выражения можно понимать и без перевода математического языка на язык человеческий.

Вернём получившееся равенство 2 = 10 − 8 в первоначальное состояние:

8 + 2 = 10

Выразим в этот раз число 8. Что нужно сделать с остальными числами, чтобы получить число 8? Верно, нужно из числа 10 вычесть число 2

8 = 10 − 2

Вернем получившееся равенство 8 = 10 − 2 в первоначальное состояние:

8 + 2 = 10

В этот раз выразим число 10. Но оказывается, что десятку выражать не нужно, поскольку она уже выражена. Достаточно поменять местами левую и правую часть, тогда получится то, что нам нужно:

10 = 8 + 2


Пример 2. Рассмотрим равенство 8 − 2 = 6

Выразим из этого равенства число 8. Чтобы выразить число 8 остальные два числа нужно сложить:

8 = 6 + 2

Вернем получившееся равенство 8 = 6 + 2 в первоначальное состояние:

8 − 2 = 6

Выразим из этого равенства число 2. Чтобы выразить число 2, нужно из 8 вычесть 6

2 = 8 − 6


Пример 3. Рассмотрим равенство 3 × 2 = 6

Выразим число 3. Чтобы выразить число 3, нужно 6 разделить 2

Вернем получившееся равенство  в первоначальное состояние:

3 × 2 = 6

Выразим из этого равенства число 2. Чтобы выразить число 2, нужно 6 разделить 3


Пример 4. Рассмотрим равенство 

Выразим из этого равенства число 15. Чтобы выразить число 15, нужно перемножить числа 3 и 5

15 = 3 × 5

Вернем получившееся равенство 15 = 3 × 5 в первоначальное состояние:

Выразим из этого равенства число 5. Чтобы выразить число 5, нужно 15 разделить 3


Правила нахождения неизвестных

Рассмотрим несколько правил нахождения неизвестных. Возможно, они вам знакомы, но не мешает повторить их ещё раз. В дальнейшем их можно будет забыть, поскольку мы научимся решать уравнения, не применяя эти правила.

Вернемся к первому примеру, который мы рассматривали в предыдущей теме, где в равенстве 8 + 2 = 10 требовалось выразить число 2.

В равенстве 8 + 2 = 10 числа 8 и 2 являются слагаемыми, а число 10 — суммой.

Чтобы выразить число 2, мы поступили следующим образом:

2 = 10 − 8

То есть из суммы 10 вычли слагаемое 8.

Теперь представим, что в равенстве 8 + 2 = 10 вместо числа 2 располагается переменная x

8 + x = 10

В этом случае равенство 8 + 2 = 10 превращается в уравнение 8 + = 10, а переменная x берет на себя роль так называемого неизвестного слагаемого

Наша задача найти это неизвестное слагаемое, то есть решить уравнение 8 + = 10. Для нахождения неизвестного слагаемого предусмотрено следующее правило:

Чтобы найти неизвестное слагаемое, нужно из суммы вычесть известное слагаемое.

Что мы в принципе и сделали, когда выражали двойку в равенстве 8 + 2 = 10. Чтобы выразить слагаемое 2, мы из суммы 10 вычли другое слагаемое 8

2 = 10 − 8

А сейчас, чтобы найти неизвестное слагаемое x, мы должны из суммы 10 вычесть известное слагаемое 8:

x = 10 − 8

Если вычислить правую часть получившегося равенства, то можно узнать чему равна переменная x

x = 2

Мы решили уравнение. Значение переменной x равно 2. Для проверки значение переменной x отправляют в исходное уравнение 8 + = 10 и подставляют вместо x. Так желательно поступать с любым решённым уравнением, поскольку нельзя быть точно уверенным, что уравнение решено правильно:

В результате получается верное числовое равенство. Значит уравнение решено правильно.

Это же правило действовало бы в случае, если неизвестным слагаемым было бы первое число 8.

x + 2 = 10

В этом уравнении x — это неизвестное слагаемое, 2 — известное слагаемое, 10 — сумма. Чтобы найти неизвестное слагаемое x, нужно из суммы 10 вычесть известное слагаемое 2

x = 10 − 2

x = 8


Вернемся ко второму примеру из предыдущей темы, где в равенстве 8 − 2 = 6 требовалось выразить число 8.

В равенстве 8 − 2 = 6 число 8 это уменьшаемое, число 2 — вычитаемое, число 6 — разность

Чтобы выразить число 8, мы поступили следующим образом:

8 = 6 + 2

То есть сложили разность 6 и вычитаемое 2.

Теперь представим, что в равенстве 8 − 2 = 6 вместо числа 8 располагается переменная x

x − 2 = 6

В этом случае переменная x берет на себя роль так называемого неизвестного уменьшаемого

Для нахождения неизвестного уменьшаемого предусмотрено следующее правило:

Чтобы найти неизвестное уменьшаемое, нужно к разности прибавить вычитаемое.

Что мы и сделали, когда выражали число 8 в равенстве 8 − 2 = 6. Чтобы выразить уменьшаемое 8, мы к разности 6 прибавили вычитаемое 2.

А сейчас, чтобы найти неизвестное уменьшаемое x, мы должны к разности 6 прибавить вычитаемое 2

x = 6 + 2

Если вычислить правую часть, то можно узнать чему равна переменная x

x = 8


Теперь представим, что в равенстве 8 − 2 = 6 вместо числа 2 располагается переменная x

8 − x = 6

В этом случае переменная x берет на себя роль неизвестного вычитаемого

Для нахождения неизвестного вычитаемого предусмотрено следующее правило:

Чтобы найти неизвестное вычитаемое, нужно из уменьшаемого вычесть разность.

Что мы и сделали, когда выражали число 2 в равенстве 8 − 2 = 6. Чтобы выразить число 2, мы из уменьшаемого 8 вычли разность 6.

А сейчас, чтобы найти неизвестное вычитаемое x, нужно опять же из уменьшаемого 8 вычесть разность 6

x = 8 − 6

Вычисляем правую часть и находим значение x

x = 2


Вернемся к третьему примеру из предыдущей темы, где в равенстве 3 × 2 = 6 мы пробовали выразить число 3.

В равенстве 3 × 2 = 6 число 3 — это множимое, число 2 — множитель, число 6 — произведение

Чтобы выразить число 3 мы поступили следующим образом:

То есть разделили произведение 6 на множитель 2.

Теперь представим, что в равенстве 3 × 2 = 6 вместо числа 3 располагается переменная x

x × 2 = 6

В этом случае переменная x берет на себя роль неизвестного множимого.

Для нахождения неизвестного множимого предусмотрено следующее правило:

Чтобы найти неизвестное множимое, нужно произведение разделить на множитель.

Что мы и сделали, когда выражали число 3 из равенства 3 × 2 = 6. Произведение 6 мы разделили на множитель 2.

А сейчас для нахождения неизвестного множимого x, нужно произведение 6 разделить на множитель 2.

Вычисление правой части позволяет нам найти значение переменной x

x = 3

Это же правило применимо в случае, если переменная x располагается вместо множителя, а не множимого. Представим, что в равенстве 3 × 2 = 6 вместо числа 2 располагается переменная x.

В этом случае переменная x берет на себя роль неизвестного множителя. Для нахождения неизвестного множителя предусмотрено такое же, что и для нахождения неизвестного множимого, а именно деление произведения на известный множитель:

Чтобы найти неизвестный множитель, нужно произведение разделить на множимое.

Что мы и сделали, когда выражали число 2 из равенства 3 × 2 = 6. Тогда для получения числа 2 мы разделили произведение 6 на множимое 3.

А сейчас для нахождения неизвестного множителя x мы разделили произведение 6 на множимое 3.

Вычисление правой части равенства  позволяет узнать чему равно x

x = 2

Множимое и множитель вместе называют сомножителями. Поскольку правила нахождения множимого и множителя совпадают, мы можем сформулировать общее правило нахождения неизвестного сомножителя:

Чтобы найти неизвестный сомножитель, нужно произведение разделить на известный сомножитель.

Например, решим уравнение 9 × x = 18. Переменная x является неизвестным сомножителем. Чтобы найти этот неизвестный сомножитель, нужно произведение 18 разделить на известный сомножитель 9

Отсюда .

Решим уравнение × 3 = 27. Переменная x является неизвестным сомножителем. Чтобы найти этот неизвестный сомножитель, нужно произведение 27 разделить на известный сомножитель 3

Отсюда .


Вернемся к четвертому примеру из предыдущей темы, где в равенстве  требовалось выразить число 15. В этом равенстве число 15 — это делимое, число 5 — делитель, число 3 — частное.

Чтобы выразить число 15 мы поступили следующим образом:

15 = 3 × 5

То есть умножили частное 3 на делитель 5.

Теперь представим, что в равенстве  вместо числа 15 располагается переменная x

В этом случае переменная x берет на себя роль неизвестного делимого.

Для нахождения неизвестного делимого предусмотрено следующее правило:

Чтобы найти неизвестное делимое, нужно частное умножить на делитель.

Что мы и сделали, когда выражали число 15 из равенства . Чтобы выразить число 15, мы умножили частное 3 на делитель 5.

А сейчас, чтобы найти неизвестное делимое x, нужно частное 3 умножить на делитель 5

x = 3 × 5

Вычислим правую часть получившегося равенства. Так мы узнаем чему равна переменная x.

x = 15


Теперь представим, что в равенстве  вместо числа 5 располагается переменная x.

В этом случае переменная x берет на себя роль неизвестного делителя.

Для нахождения неизвестного делителя предусмотрено следующее правило:

Чтобы найти неизвестный делитель, нужно делимое разделить на частное.

Что мы и сделали, когда выражали число 5 из равенства .  Чтобы выразить число 5, мы разделили делимое 15 на частное 3.

А сейчас, чтобы найти неизвестный делитель x, нужно делимое 15 разделить на частное 3

Вычислим правую часть получившегося равенства. Так мы узнаем чему равна переменная x.

x = 5

Итак, для нахождения неизвестных мы изучили следующие правила:

  • Чтобы найти неизвестное слагаемое, нужно из суммы вычесть известное слагаемое;
  • Чтобы найти неизвестное уменьшаемое, нужно к разности прибавить вычитаемое;
  • Чтобы найти неизвестное вычитаемое, нужно из уменьшаемого вычесть разность;
  • Чтобы найти неизвестное множимое, нужно произведение разделить на множитель;
  • Чтобы найти неизвестный множитель, нужно произведение разделить на множимое;
  • Чтобы найти неизвестное делимое, нужно частное умножить на делитель;
  • Чтобы найти неизвестный делитель, нужно делимое разделить на частное.

Компоненты

Компонентами мы будем называть числа и переменные, входящие в равенство

Так, компонентами сложения являются слагаемые и сумма


Компонентами вычитания являются уменьшаемое, вычитаемое и разность


Компонентами умножения являются множимое, множитель и произведение


Компонентами деления являются делимое, делитель и частное

В зависимости от того, с какими компонентами мы будем иметь дело, будут применяться соответствующие правила нахождения неизвестных. Эти правила мы изучили в предыдущей теме. При решении уравнений желательно знать эти правило наизусть.

Пример 1. Найти корень уравнения 45 + x = 60

45 — слагаемое, x — неизвестное слагаемое, 60 — сумма. Имеем дело с компонентами сложения. Вспоминаем, что для нахождения неизвестного слагаемого, нужно из суммы вычесть известное слагаемое:

x = 60 − 45

Вычислим правую часть, получим значение x равное 15

x = 15

Значит корень уравнения 45 + x = 60 равен 15.

Чаще всего неизвестное слагаемое необходимо привести к виду при котором его можно было бы выразить.

Пример 2. Решить уравнение 

Здесь в отличие от предыдущего примера, неизвестное слагаемое нельзя выразить сразу, поскольку оно содержит коэффициент 2. Наша задача привести это уравнение к виду при котором можно было бы выразить x

В данном примере мы имеем дело с компонентами сложения — слагаемыми и суммой. 2x — это первое слагаемое, 4 — второе слагаемое, 8 — сумма.

При этом слагаемое 2x содержит переменную x. После нахождения значения переменной x слагаемое 2x примет другой вид. Поэтому слагаемое 2x можно полностью принять за неизвестное слагаемое:

Теперь применяем правило нахождения неизвестного слагаемого. Вычитаем из суммы известное слагаемое:

Вычислим правую часть получившегося уравнения:

Мы получили новое уравнение . Теперь мы имеем дело с компонентами умножения: множимым, множителем и произведением. 2 — множимое, — множитель, 4 — произведение

При этом переменная x является не просто множителем, а неизвестным множителем

Чтобы найти этот неизвестный множитель, нужно произведение разделить на множимое:

Вычислим правую часть, получим значение переменной x

Для проверки найденный корень отправим в исходное уравнение  и подставим вместо x

Получили верное числовое равенство. Значит уравнение решено правильно.


Пример 3. Решить уравнение 3+ 9+ 16= 56

Cразу выразить неизвестное x нельзя. Сначала нужно привести данное уравнение к виду при котором его можно было бы выразить.

Приведем подобные слагаемые в левой части данного уравнения:

Имеем дело с компонентами умножения. 28 — множимое, — множитель, 56 — произведение. При этом x является неизвестным множителем. Чтобы найти неизвестный множитель, нужно произведение разделить на множимое:

Отсюда x равен 2


Равносильные уравнения

В предыдущем примере при решении уравнения 3x + 9x + 16x = 56, мы привели подобные слагаемые в левой части уравнения. В результате получили новое уравнение 28x = 56. Старое уравнение 3x + 9x + 16x = 56 и получившееся новое уравнение 28x = 56 называют равносильными уравнениями, поскольку их корни совпадают.

Уравнения называют равносильными, если их корни совпадают.

Проверим это. Для уравнения 3+ 9+ 16= 56 мы нашли корень равный 2. Подставим этот корень сначала в уравнение 3+ 9+ 16= 56, а затем в уравнение 28= 56, которое получилось в результате приведения подобных слагаемых в левой части предыдущего уравнения. Мы должны получить верные числовые равенства

Согласно порядку действий, в первую очередь выполняется умножение:

Подставим корень 2 во второе уравнение 28= 56

Видим, что у обоих уравнений корни совпадают. Значит уравнения 3+ 9+ 16= 56 и 28= 56 действительно являются равносильными.

Для решения уравнения 3+ 9+ 16= 56 мы воспользовались одним из тождественных преобразований — приведением подобных слагаемых. Правильное тождественное преобразование уравнения позволило нам получить равносильное уравнение 28= 56, которое проще решать.

Из тождественных преобразований на данный момент мы умеем только сокращать дроби, приводить подобные слагаемые, выносить общий множитель за скобки, а также раскрывать скобки. Существуют и другие преобразования, которые следует знать. Но для общего представления о тождественных преобразованиях уравнений, изученных нами тем вполне хватает.


Рассмотрим некоторые преобразования, которые позволяют получить равносильное уравнение

Если к обеим частям уравнения прибавить одно и то же число, то получится уравнение равносильное данному.

и аналогично:

Если из обеих частей уравнения вычесть одно и то же число, то получится уравнение равносильное данному.

Другими словами, корень уравнения не изменится, если к обеим частям данного уравнения прибавить (или вычесть из обеих частей) одно и то же число.

Пример 1. Решить уравнение

Вычтем из обеих частей уравнения число 10

Приведем подобные слагаемые в обеих частях:

Получили уравнение 5= 10. Имеем дело с компонентами умножения. Чтобы найти неизвестный сомножитель x, нужно произведение 10 разделить на известный сомножитель 5.

Отсюда .

Вернемся к исходному уравнению  и подставим вместо x найденное значение 2

Получили верное числовое равенство. Значит уравнение решено правильно.

Решая уравнение мы вычли из обеих частей уравнения число 10. В результате получили равносильное уравнение . Корень этого уравнения, как и уравнения  так же равен 2


Пример 2. Решить уравнение 4(+ 3) = 16

Раскроем скобки в левой части равенства:

Вычтем из обеих частей уравнения число 12

Приведем подобные слагаемые в обеих частях уравнения:

В левой части останется 4x, а в правой части число 4

 

 

Получили уравнение 4= 4. Имеем дело с компонентами умножения. Чтобы найти неизвестный сомножитель x, нужно произведение 4 разделить на известный сомножитель 4

Отсюда 

Вернемся к исходному уравнению 4(+ 3) = 16 и подставим вместо x найденное значение 1

 

Получили верное числовое равенство. Значит уравнение решено правильно.

Решая уравнение 4(+ 3) = 16 мы вычли из обеих частей уравнения число 12. В результате получили равносильное уравнение 4= 4. Корень этого уравнения, как и уравнения 4(+ 3) = 16 так же равен 1


Пример 3. Решить уравнение

Раскроем скобки в левой части равенства:

Прибавим к обеим частям уравнения число 8

Приведем подобные слагаемые в обеих частях уравнения:

В левой части останется 2x, а в правой части число 9

В получившемся уравнении 2= 9 выразим неизвестное слагаемое x

 

Отсюда 

Вернемся к исходному уравнению  и подставим вместо x найденное значение 4,5

Получили верное числовое равенство. Значит уравнение решено правильно.

Решая уравнение  мы прибавили к обеим частям уравнения число 8. В результате получили равносильное уравнение . Корень этого уравнения, как и уравнения  так же равен 4,5


Следующее правило, которое позволяет получить равносильное уравнение, выглядит следующим образом

Если в уравнении перенести слагаемое из одной части в другую, изменив его знак, то получится уравнение равносильное данному.

То есть корень уравнения не изменится, если мы перенесем слагаемое из одной части уравнения в другую, изменив его знак. Это свойство является одним из важных и одним из часто используемых при решении уравнений.

Рассмотрим следующее уравнение:

Корень данного уравнения равен 2. Подставим вместо x этот корень и проверим получается ли верное числовое равенство

Получается верное равенство. Значит число 2 действительно является корнем уравнения .

Теперь попробуем поэкспериментировать со слагаемыми этого уравнения, перенося их из одной части в другую, изменяя знаки.

Например, слагаемое 3x располагается в левой части равенства. Перенесём его в правую часть, изменив знак на противоположный:

Получилось уравнение 12 = 9x − 3x. Приведем подобные слагаемые в правой части данного уравнения:

Имеем дело с компонентами умножения. Переменная x является неизвестным сомножителем. Найдём этот известный сомножитель:

Отсюда = 2. Как видим, корень уравнения не изменился. Значит уравнения 12 + 3x = 9x и 12 = 9x − 3x являются равносильными.

На самом деле данное преобразование является упрощенным методом предыдущего преобразования, где к обеим частям уравнения прибавлялось (или вычиталось) одно и то же число.

Мы сказали, что в уравнении 12 + 3x = 9x слагаемое 3x было перенесено в правую часть, изменив знак. В реальности же происходило следующее: из обеих частей уравнения вычли слагаемое 3x

Затем в левой части были приведены подобные слагаемые и получено уравнение 12 = 9x − 3x. Затем опять были приведены подобные слагаемые, но уже в правой части, и получено уравнение 12 = 6x.

Но так называемый «перенос» более удобен для подобных уравнений, поэтому он и получил такое широкое распространение. Решая уравнения, мы часто будем пользоваться именно этим преобразованием.

Равносильными также являются уравнения 12 + 3= 9x и 3x − 9= −12. В этот раз в уравнении 12 + 3= 9x слагаемое 12 было перенесено в правую часть, а слагаемое 9x в левую. Не следует забывать, что знаки этих слагаемых были изменены во время переноса


Следующее правило, которое позволяет получить равносильное уравнение, выглядит следующим образом:

Если обе части уравнения умножить или разделить на одно и то же число, не равное нулю, то получится уравнение равносильное данному.

Другими словами, корни уравнения не изменятся, если обе его части умножить или разделить на одно и то же число. Это действие часто применяется тогда, когда нужно решить уравнение содержащее дробные выражения.

Сначала рассмотрим примеры, в которых обе части уравнения будут умножаться на одно и то же число.

Пример 1. Решить уравнение 

При решении уравнений, содержащих дробные выражения, сначала  принято упростить это уравнение.

В данном случае мы имеем дело именно с таким уравнением. В целях упрощения данного уравнения обе его части можно умножить на 8:

Мы помним, что для умножения дроби на число, нужно числитель данной дроби умножить на это число. У нас имеются две дроби и каждая из них умножается на число 8. Наша задача умножить числители дробей на это число 8

Теперь происходит самое интересное. В числителях и знаменателях обеих дробей содержится множитель 8, который можно сократить на 8. Это позволит нам избавиться от дробного выражения:

В результате останется простейшее уравнение

Ну и нетрудно догадаться, что корень этого уравнения равен 4

Вернемся к исходному уравнению   и подставим вместо x найденное значение 4

Получается верное числовое равенство. Значит уравнение решено правильно.

При решении данного уравнения мы умножили обе его части на 8. В результате получили уравнение . Корень этого уравнения, как и уравнения  равен 4. Значит эти уравнения равносильны.

Множитель на который умножаются обе части уравнения принято записывать перед частью уравнения, а не после неё. Так, решая уравнение , мы умножили обе части на множитель 8 и получили следующую запись:

От этого корень уравнения не изменился, но если бы мы сделали это находясь в школе, то нам сделали бы замечание, поскольку в алгебре множитель принято записывать перед тем выражением, с которым он перемножается. Поэтому умножение обеих частей уравнения  на множитель 8 желательно переписать следующим образом:


Пример 2. Решить уравнение 

Умнóжим обе части уравнения на 15

В левой части множители 15 можно сократить на 15, а в правой части множители 15 и 5 можно сократить на 5

Перепишем то, что у нас осталось:

Раскроем скобки в правой части уравнения:

Перенесем слагаемое x из левой части уравнения в правую часть, изменив знак. А слагаемое 15 из правой части уравнения перенесем в левую часть, опять же изменив знак:

Приведем подобные слагаемые в обеих частях, получим

Имеем дело с компонентами умножения. Переменная x является неизвестным сомножителем. Найдём этот известный сомножитель:

Отсюда 

Вернемся к исходному уравнению   и подставим вместо найденное значение 5

Получается верное числовое равенство. Значит уравнение решено правильно. При решении данного уравнения мы умножили обе го части на 15. Далее выполняя тождественные преобразования, мы получили уравнение 10 = 2x. Корень этого уравнения, как и уравнения  равен 5. Значит эти уравнения равносильны.


Пример 3. Решить уравнение 

Умнóжим обе части уравнения на 3

В левой части можно сократить две тройки, а правая часть будет равна 18

Останется простейшее уравнение . Имеем дело с компонентами умножения. Переменная x является неизвестным сомножителем. Найдём этот известный сомножитель:

Отсюда 

Вернемся к исходному уравнению   и подставим вместо найденное значение 9

Получается верное числовое равенство. Значит уравнение решено правильно.


Пример 4. Решить уравнение 

Умнóжим обе части уравнения на 6

В левой части уравнения раскроем скобки. В правой части множитель 6 можно поднять в числитель:

Сократим в обеих частях уравнениях то, что можно сократить:

Перепишем то, что у нас осталось:

Раскроем скобки в обеих частях уравнения:

Воспользуемся переносом слагаемых. Слагаемые, содержащие неизвестное x, сгруппируем в левой части уравнения, а слагаемые свободные от неизвестных — в правой:

Приведем подобные слагаемые в обеих частях:

Теперь найдем значение переменной x. Для этого разделим произведение 28 на известный сомножитель 7

Отсюда = 4.

Вернемся к исходному уравнению  и подставим вместо x найденное значение 4

Получилось верное числовое равенство. Значит уравнение решено правильно.


Пример 5. Решить уравнение 

Раскроем скобки в обеих частях уравнения там, где это можно:

Умнóжим обе части уравнения на 15

Раскроем скобки в обеих частях уравнения:

Сократим в обеих частях уравнения, то что можно сократить:

Перепишем то, что у нас осталось:

Раскроем скобки там, где это можно:

Воспользуемся переносом слагаемых. Слагаемые, содержащие неизвестное, сгруппируем в левой части уравнения, а слагаемые, свободные от неизвестных — в правой. Не забываем, что во время переноса, слагаемые меняют свои знаки на противоположные:

Приведем подобные слагаемые в обеих частях уравнения:

Найдём значение x

В получившемся ответе можно выделить целую часть:

Вернемся к исходному уравнению и подставим вместо x найденное значение 

Получается довольно громоздкое выражение. Воспользуемся переменными. Левую часть равенства занесем в переменную A, а правую часть равенства в переменную B

Наша задача состоит в том, чтобы убедиться равна ли левая часть правой. Другими словами, доказать равенство A = B

Найдем значение выражения, находящегося в переменной А.

Значение переменной А равно . Теперь найдем значение переменной B. То есть значение правой части нашего равенства. Если и оно равно , то уравнение будет решено верно

Видим, что значение переменной B, как и значение переменной A равно . Это значит, что левая часть равна правой части. Отсюда делаем вывод, что уравнение решено правильно.

Теперь попробуем не умножать обе части уравнения на одно и то же число, а делить.

Рассмотрим уравнение 30+ 14+ 14 = 70− 40+ 42. Решим его обычным методом: слагаемые, содержащие неизвестные, сгруппируем в левой части уравнения, а слагаемые, свободные от неизвестных — в правой. Далее выполняя известные тождественные преобразования, найдем значение x

Подставим найденное значение 2 вместо x в исходное уравнение:

Теперь попробуем разделить все слагаемые уравнения 30+ 14+ 14 = 70− 40+ 42 на какое-нибудь число. Замечаем, что все слагаемые этого уравнения имеют общий множитель 2. На него и разделим каждое слагаемое:

Выполним сокращение в каждом слагаемом:

Перепишем то, что у нас осталось:

Решим это уравнение, пользуясь известными тождественными преобразованиями:

Получили корень 2. Значит уравнения 15+ 7+ 7 = 35x − 20+ 21 и 30+ 14+ 14 = 70− 40+ 42 равносильны.

Деление обеих частей уравнения на одно и то же число позволяет освобождать неизвестное от коэффициента. В предыдущем примере когда мы получили уравнение 7= 14, нам потребовалось разделить произведение 14 на известный сомножитель 7. Но если бы мы в левой части освободили неизвестное от коэффициента 7, корень нашелся бы сразу. Для этого достаточно было разделить обе части на 7

Этим методом мы тоже будем пользоваться часто.


Умножение на минус единицу

Если обе части уравнения умножить на минус единицу, то получится уравнение равносильное данному.

Это правило следует из того, что от умножения (или деления) обеих частей уравнения на одно и то же число, корень данного уравнения не меняется. А значит корень не поменяется если обе его части умножить на −1.

Данное правило позволяет поменять знаки всех компонентов, входящих в уравнение. Для чего это нужно? Опять же, чтобы получить равносильное уравнение, которое проще решать.

Рассмотрим уравнение . Чему равен корень этого уравнения?

Прибавим к обеим частям уравнения число 5

Приведем подобные слагаемые:

А теперь вспомним про коэффициент буквенного выражения. Что же представляет собой левая часть уравнения . Это есть произведение минус единицы и переменной x

То есть минус, стоящий перед переменной x, относится не к самой переменной x, а к единице, которую мы не видим, поскольку коэффициент 1 принято не записывать. Это означает, что уравнение  на самом деле выглядит следующим образом:

Имеем дело с компонентами умножения. Чтобы найти х, нужно произведение −5 разделить на известный сомножитель −1.

или разделить обе части уравнения на −1, что еще проще

Итак, корень уравнения  равен 5. Для проверки подставим его в исходное уравнение. Не забываем, что в исходном уравнении минус стоящий перед переменной x относится к невидимой единице

Получилось верное числовое равенство. Значит уравнение решено верно.

Теперь попробуем умножить обе части уравнения  на минус единицу:

После раскрытия скобок в левой части образуется выражение , а правая часть будет равна 10

Корень этого уравнения, как и уравнения  равен 5

Значит уравнения  и  равносильны.


Пример 2. Решить уравнение 

В данном уравнении все компоненты являются отрицательными. С положительными компонентами работать удобнее, чем с отрицательными, поэтому поменяем знаки всех компонентов, входящих в уравнение . Для этого умнóжим обе части данного уравнения на −1.

Понятно, что от умножения на −1 любое число поменяет свой знак на противоположный. Поэтому саму процедуру умножения на −1 и раскрытие скобок подробно не расписывают, а сразу записывают компоненты уравнения с противоположными знаками.

Так, умножение уравнения  на −1 можно записать подробно следующим образом:

либо можно просто поменять знаки всех компонентов:

Получится то же самое, но разница будет в том, что мы сэкономим себе время.

Итак, умножив обе части уравнения  на −1, мы получили уравнение . Решим данное уравнение. Из обеих частей вычтем число 4 и разделим обе части на 3

Когда корень найден, переменную обычно записывают в левой части, а её значение в правой, что мы и сделали.


Пример 3. Решить уравнение 

Умнóжим обе части уравнения на −1. Тогда все компоненты поменяют свои знаки на противоположные:

Из обеих частей получившегося уравнения вычтем 2x и приведем подобные слагаемые:

Прибавим к обеим частям уравнения единицу и приведем подобные слагаемые: 


Приравнивание к нулю

Недавно мы узнали, что если в уравнении перенести слагаемое из одной части в другую, изменив его знак, то получится уравнение равносильное данному.

А что будет если перенести из одной части в другую не одно слагаемое, а все слагаемые? Верно, в той части откуда забрали все слагаемые останется ноль. Иными словами, не останется ничего.

В качестве примера рассмотрим уравнение . Решим данное уравнение, как обычно — слагаемые, содержащие неизвестные сгруппируем в одной части, а числовые слагаемые, свободные от неизвестных оставим в другой. Далее выполняя известные тождественные преобразования, найдем значение переменной x

Теперь попробуем решить это же уравнение, приравняв все его компоненты к нулю. Для этого перенесем все слагаемые из правой части в левую, изменив знаки:

Приведем подобные слагаемые в левой части:

Прибавим к обеим частям 77, и разделим обе части на 7


Альтернатива правилам нахождения неизвестных

Очевидно, что зная о тождественных преобразованиях уравнений, можно не заучивать наизусть правила нахождения неизвестных.

К примеру, для нахождения неизвестного в уравнении  мы произведение 10 делили на известный сомножитель 2

Но если в уравнении  обе части разделить на 2 корень найдется сразу. В левой части уравнения в числителе множитель 2 и в знаменателе множитель 2 сократятся на 2. А правая часть будет  равна 5

Уравнения вида  мы решали выражая неизвестное слагаемое:

Но можно воспользоваться тождественными преобразованиями, которые мы сегодня изучили. В уравнении слагаемое 4 можно перенести в правую часть, изменив знак:

Далее разделить обе части на 2

В левой части уравнения сократятся две двойки. Правая часть будет равна 2. Отсюда .

Либо можно было из обеих частей уравнения вычесть 4. Тогда получилось бы следующее:

В случае с уравнениями вида  удобнее делить произведение на известный сомножитель. Сравним оба решения:

Первое решение намного короче и аккуратнее. Второе решение можно значительно укоротить, если выполнить деление в уме.

Тем не менее, необходимо знать оба метода, и только затем использовать тот, который больше нравится.


Когда корней несколько

Уравнение может иметь несколько корней. Например уравнение x(x + 9) = 0 имеет два корня: 0 и −9.

В уравнении x(x + 9) = 0 нужно было найти такое значение при котором левая часть была бы равна нулю. В левой части этого уравнения содержатся выражения x и (x + 9), которые являются сомножителями. Из законов умножения мы знаем, что произведение равно нулю, если хотя бы один из сомножителей равен нулю (или первый сомножитель или второй).

То есть в уравнении x(x + 9) = 0 равенство будет достигаться, если x будет равен нулю или (x + 9) будет равно нулю.

x = 0 или x + 9 = 0

Приравняв к нулю оба этих выражения, мы сможем найти корни уравнения x(x + 9) = 0. Первый корень, как видно из примера, нашелся сразу. Для нахождения второго корня нужно решить элементарное уравнение + 9 = 0. Несложно догадаться, что корень этого уравнения равен −9. Проверка показывает, что корень верный:

−9 + 9 = 0


Пример 2. Решить уравнение

Данное уравнение имеет два корня: 1 и 2. Левая часть уравнения является произведение выражений (x − 1) и (x − 2). А произведение равно нулю, если хотя бы один из сомножителей равен нулю (или сомножитель (x − 1) или сомножитель (x − 2)).

Найдем такое x при котором выражения (x − 1) или (x − 2) обращаются в нули:

Подставляем по-очереди найденные значения в исходное уравнение  и убеждаемся, что при этих значениях левая часть равняется нулю:


Когда корней бесконечно много

Уравнение может иметь бесконечно много корней. То есть подставив в такое уравнение любое число, мы получим верное числовое равенство.

Пример 1. Решить уравнение 

Корнем данного уравнения является любое число. Если раскрыть скобки в левой части уравнения и привести подобные слагаемые, то получится равенство 14 = 14. Это равенство будет получаться при любом x


Пример 2. Решить уравнение 

Корнем данного уравнения является любое число. Если раскрыть скобки в левой части уравнения, то получится равенство 10x + 12 = 10x + 12. Это равенство будет получаться при любом x


Когда корней нет

Случается и так, что уравнение вовсе не имеет решений, то есть не имеет корней. Например уравнение не имеет корней, поскольку при любом значении x, левая часть уравнения не будет равна правой части. Например, пусть . Тогда уравнение примет следующий вид

Пусть


Пример 2. Решить уравнение 

Раскроем скобки в левой части равенства:

Приведем подобные слагаемые:

Видим, что левая часть не равна правой части. И так будет при любом значении y. Например, пусть y = 3.


Буквенные уравнения

Уравнение может содержать не только числа с переменными, но и буквы.

Например, формула нахождения скорости является буквенным уравнением:

Данное уравнение описывает скорость движения тела при равноускоренном движении.

Полезным навыком является умение выразить любой компонент, входящий в буквенное уравнение. Например, чтобы из уравнения  определить расстояние, нужно выразить переменную s.

Умнóжим обе части уравнения  на t

В правой части переменные t сократим на t и перепишем то, что у нас осталось:

В получившемся уравнении левую и правую часть поменяем местами:

У нас получилась формула нахождения расстояния, которую мы изучали ранее.

Попробуем из уравнения  определить время. Для этого нужно выразить переменную t.

Умнóжим обе части уравнения на t

В правой части переменные t сократим на t и перепишем то, что у нас осталось:

В получившемся уравнении v × t = s обе части разделим на v

В левой части переменные v сократим на v и перепишем то, что у нас осталось:

У нас получилась формула определения времени, которую мы изучали ранее.

Предположим, что скорость поезда равна 50 км/ч

v = 50 км/ч

А расстояние равно 100 км

s = 100 км

Тогда буквенное уравнение примет следующий вид

Из этого уравнения можно найти время. Для этого нужно суметь выразить переменную t. Можно воспользоваться правилом нахождения неизвестного делителя, разделив делимое на частное и таким образом определить значение переменной t

либо можно воспользоваться тождественными преобразованиями. Сначала умножить обе части уравнения на t

Затем разделить обе части на 50


Пример 2. Дано буквенное уравнение . Выразите из данного уравнения x

Вычтем из обеих частей уравнения a

Разделим обе части уравнения на b

Теперь, если нам попадется уравнение вида a + bx = c, то у нас будет готовое решение. Достаточно будет подставить в него нужные значения. Те значения, которые будут подставляться вместо букв a, b, c принято называть параметрами. А уравнения вида a + bx = c называют уравнением с параметрами. В зависимости от параметров, корень будет меняться.

Решим уравнение 2 + 4x = 10. Оно похоже на буквенное уравнение a + bx = c.  Вместо того, чтобы выполнять тождественные преобразования, мы можем воспользоваться готовым решением. Сравним оба решения:

Видим, что второе решение намного проще и короче.

Для готового решения необходимо сделать небольшое замечание. Параметр b не должен быть равным нулю (b ≠ 0), поскольку деление на ноль на допускается.


Пример 3. Дано буквенное уравнение . Выразите из данного уравнения x

Раскроем скобки в обеих частях уравнения

Воспользуемся переносом слагаемых. Параметры, содержащие переменную x, сгруппируем в левой части уравнения, а параметры свободные от этой переменной — в правой.

В левой части вынесем за скобки множитель x

Разделим обе части на выражение a − b

В левой части числитель и знаменатель можно сократить на a − b. Так окончательно выразится переменная x

Теперь, если нам попадется уравнение вида a(x − c) = b(x + d), то у нас будет готовое решение. Достаточно будет подставить в него нужные значения.

Допустим нам дано уравнение 4(x − 3) = 2(+ 4). Оно похоже на уравнение a(x − c) = b(x + d). Решим его двумя способами: при помощи тождественных преобразований и при помощи готового решения:

Для удобства вытащим из уравнения 4(x − 3) = 2(+ 4) значения параметров a, b, c, d. Это позволит нам не ошибиться при подстановке:

Как и в прошлом примере знаменатель здесь не должен быть равным нулю (a − b ≠ 0). Если нам встретится уравнение вида a(x − c) = b(x + d) в котором параметры a и b будут одинаковыми, мы сможем не решая его сказать, что у данного уравнения корней нет, поскольку разность одинаковых чисел равна нулю.

Например, уравнение 2(x − 3) = 2(x + 4) является уравнением вида a(x − c) = b(x + d). В уравнении 2(x − 3) = 2(x + 4) параметры a и b одинаковые. Если мы начнём его решать, то придем к тому, что левая часть не будет равна правой части:


Пример 4. Дано буквенное уравнение . Выразите из данного уравнения x

Приведем левую часть уравнения к общему знаменателю:

Умнóжим обе части на a

В левой части x вынесем за скобки

Разделим обе части на выражение (1 − a)


Линейные уравнения с одним неизвестным

Рассмотренные в данном уроке уравнения называют линейными уравнениями первой степени с одним неизвестным.

Если уравнение дано в первой степени, не содержит деления на неизвестное, а также не содержит корней из неизвестного, то его можно назвать линейным. Мы еще не изучали степени и корни, поэтому чтобы не усложнять себе жизнь, слово «линейный» будем понимать как «простой».

Большинство уравнений, решенных в данном уроке, в конечном итоге сводились к простейшему уравнению, в котором нужно было произведение разделить на известный сомножитель. Таковым к примеру является уравнение 2(x + 3) = 16. Давайте решим его.

Раскроем скобки в левой части уравнения, получим 2+ 6 = 16. Перенесем слагаемое 6 в правую часть, изменив знак. Тогда получим 2= 16 − 6. Вычислим правую часть, получим 2= 10. Чтобы найти x, разделим произведение 10 на известный сомножитель 2. Отсюда x = 5.

Уравнение 2(x + 3) = 16 является линейным. Оно свелось к уравнению 2= 10, для нахождения корня которого потребовалось разделить произведение на известный сомножитель. Такое простейшее уравнение называют линейным уравнением первой степени с одним неизвестным в каноническом виде. Слово «канонический» является синонимом слов «простейший» или «нормальный».

Линейное уравнение первой степени с одним неизвестным в каноническом виде называют уравнение вида ax = b.

Полученное нами уравнение 2= 10 является линейным уравнением первой степени с одним неизвестным в каноническом виде. У этого уравнения первая степень, одно неизвестное, оно не содержит деления на неизвестное и не содержит корней из неизвестного, и представлено оно в каноническом виде, то есть в простейшем виде при котором легко можно определить значение x. Вместо параметров a и b в нашем уравнении содержатся числа 2 и 10. Но подобное уравнение может содержать и другие числа: положительные, отрицательные или равные нулю.

Если в линейном уравнении a = 0 и b = 0, то уравнение имеет бесконечно много корней. Действительно, если a равно нулю и b равно нулю, то линейное уравнение ax b примет вид 0= 0. При любом значении x левая часть будет равна правой части.

Если в линейном уравнении a = 0 и b ≠ 0, то уравнение корней не имеет. Действительно, если a равно нулю и b равно какому-нибудь числу, не равному нулю, скажем числу 5, то уравнение ax = b примет вид 0= 5. Левая часть будет равна нулю, а правая часть пяти. А ноль не равен пяти.

Если в линейном уравнении a ≠ 0, и b равно любому числу, то уравнение имеет один корень. Он определяется делением параметра b на параметр a

Действительно, если a равно какому-нибудь числу, не равному нулю, скажем числу 3, и b равно какому-нибудь числу, скажем числу 6, то уравнение  примет вид .
Отсюда .

Существует и другая форма записи линейного уравнения первой степени с одним неизвестным. Выглядит она следующим образом: ax − b = 0. Это то же самое уравнение, что и ax = b, но параметр b перенесен в левую часть с противоположным знаком. Такие уравнение мы тоже решали в данном уроке. Например, уравнение 7− 77 = 0. Уравнение вида ax − b = 0 называют линейным уравнением первой степени с одним неизвестным в общем виде.

В будущем после изучения рациональных выражений, мы рассмотрим такие понятия, как посторонние корни и потеря корней. А пока рассмотренного в данном уроке будет достаточным.

Задания для самостоятельного решения

Задание 1. Используя метод переноса слагаемого, решите следующее уравнение:

Задание 2. Используя метод прибавления (или вычитания) числа к обеим частям, решите следующее уравнение:

Задание 3. Решите уравнение:

Задание 4. Решите уравнение:

Задание 5. Решите уравнение:

Задание 6. Решите уравнение:

Задание 7. Решите уравнение:

Задание 8. Решите уравнение:

Задание 9. Решите уравнение:

Задание 10. Решите уравнение:

Задание 11. Решите уравнение:

Задание 12. Решите уравнение:

Задание 13. Решите уравнение:

Задание 14. Решите уравнение:

Задание 15. Решите уравнение:

Задание 16. Решите уравнение:

Задание 17. Решите уравнение:

Задание 18. Решите уравнение:

Задание 19. Решите уравнение:

Задание 20. Решите уравнение:

Задание 21. Решите уравнение:

Задание 22. Решите уравнение:

Задание 23. Решите уравнение:

Задание 24. Решите уравнение:

Задание 25. Решите уравнение:

Задание 26. Решите уравнение:

Задание 27. Решите уравнение:

Задание 28. Решите уравнение:

Задание 29. Решите уравнение:

Задание 30. Решите уравнение:

Задание 31. Решите уравнение:

Задание 32. В следующем буквенном уравнении выразите переменную x:

Задание 33. В следующем буквенном уравнении выразите переменную x:

Задание 34. В следующем буквенном уравнении выразите переменную x:

Задание 35. В следующем буквенном уравнении выразите переменную x:

Задание 36. В следующем буквенном уравнении выразите переменную y:

Задание 37. В следующем буквенном уравнении выразите переменную z:


Понравился урок?
Вступай в нашу новую группу Вконтакте и начни получать уведомления о новых уроках



Возникло желание поддержать проект?
Используй кнопку ниже

Навигация по записям

Урок 13.2 = a

319

Условие:

Имеет ли корни уравнение:

Решение:

Советы:

Число в квадрате не может быть отрицательным

320

Условие:

Решите уравнение:

Решение:

Советы:

Квадратное уравнение может иметь два корня

321

Условие:

Решите уравнение и с помощью графика функции 

найдите приближенные значения его корней

Решение:

Советы:

Найди значения х по графику

322

Условие:

Решите уравнение:

Решение:

Советы:

Квадратное уравнение может иметь два корня, а так же не иметь корней вообще

323

Условие:

Найдите корни уравнения:

Решение:

Советы:

Квадратное уравнение может иметь два корня, а так же не иметь корней вообще

324

Условие:

Решите уравнение:

Решение:

Советы:

Квадратное уравнение может иметь два корня, а так же не иметь корней вообще

325

Условие:

Имеет ли смысл выражение 8-5х при x=-3,4; 0; 1,2; 1,6; 2,4

Решение:

Советы:

Подставь различные значения х

326

Условие:

При каких значениях переменной имеет смысл выражение:

Решение:

Советы:

Выражение под корнем должно иметь положительное значение

327

Условие:

При каких значениях переменной х имеет смысл выражение:

Решение:

Советы:

Корень из отрицательного числа не существует

328

Условие:

Найдите квадрат числа:

Решение:

Советы:

Возведи в квадрат число

329

Условие:

Найдите значение выражения:

Решение:

Советы:

отрицательное число в квадрате становится положительным

330

Условие:

Вычислите:

Решение:

Советы:

Соблюдай порядок действий

331

Условие:

Вычислите:

Решение:

Советы:

Соблюдай порядок действий

332

Условие:

Найдите значение выражения:

Решение:

Советы:

Соблюдай порядок действий

333

Условие:

Найдите значение выражения xx при x=-8; -5; 1; 7; 128. Чему равно значение выражения xx если:

а) х>0 б)  x<0

Решение:

Советы:

Вспомни свойства модуля

334

Условие:

Найдите значение выражения:

Решение:

Советы:

Соблюдай порядок действий, обращай внимание на знаки

335

Условие:

Изобразите схематически в одной и той же системе координат графики функций у = 10/x и у = 10х. Имеют ли эти графики общие точки и если имеют, то сколько?

Решение:

Советы:

Составь таблицу значений x и y, и построй график

Решите линейные уравнения с одним неизвестным x / 8-5 = -1 Tiger Algebra Solver

Переставьте:

Переставьте уравнение, вычтя то, что находится справа от знака равенства, из обеих частей уравнения:

x / 8 -5 — (- 1) = 0

Пошаговое решение:

Шаг 1:

 x
 Упростить -
            8
 
Уравнение в конце шага 1:
 x
  (- - 5) - -1 = 0
   8
 

Шаг 2:

Переписывание целого как эквивалентной дроби:

2.1 Вычитание целого из дроби

Перепишем целое как дробь, используя 8 в качестве знаменателя:

 5 5 • 8
    5 = - = —————
         1 8
 

Эквивалентная дробь: Полученная таким образом дробь выглядит иначе, но имеет то же значение, что и целое

Общий знаменатель: Эквивалентная дробь и другая дробь, участвующие в вычислении, имеют один и тот же знаменатель

 
Сложение дробей, имеющих общий знаменатель:
 

2.2 Сложение двух эквивалентных дробей
Сложите две эквивалентные дроби, которые теперь имеют общий знаменатель

Объедините числители вместе, сложите сумму или разность над общим знаменателем, затем уменьшите до наименьших членов, если возможно:

 x - (5 • 8) х - 40
 знак равно
      8 8
 
Уравнение в конце шага 2:
 (x - 40)
  ———————— - -1 = 0
     8
 

Шаг 3:

 
Переписывание целого как эквивалентной дроби:
 

3.1 Вычитание целого из дроби

Перепишем целое как дробь, используя 8 в качестве знаменателя:

 -1 -1 • 8
    -1 = —— = ——————
          1 8
 
Сложение дробей с общим знаменателем:
 

3.2 Сложение двух эквивалентных дробей

 (x-40) - (-1 • 8) x - 32
 знак равно
         8 8
 
Уравнение в конце шага 3:
 x - 32
  —————— = 0
    8
 

Шаг 4:

 
Когда дробь равна нулю:
 4.1 Когда дробь равна нулю ... 

Если дробь равна нулю, ее числитель, часть, которая находится над чертой дроби, должен быть равен нулю.

Теперь, чтобы избавиться от знаменателя, Тигр умножает обе части уравнения на знаменатель.

Вот как:

 x-32
  ———— • 8 = 0 • 8
   8
 

Теперь, с левой стороны, 8 отменяет знаменатель, в то время как с правой стороны ноль, умноженный на что-либо, все еще равно нулю.

Уравнение теперь принимает форму:
x-32 = 0

Решение уравнения с одной переменной:

4.2 Решите: x-32 = 0

Добавьте 32 к обеим сторонам уравнения:
x = 32

Было найдено одно решение:

x = 32

Решение уравнений с удалением дробей

Результаты обучения

  • Используйте наименьший общий знаменатель, чтобы исключить дроби из линейного уравнения перед его решением
  • Решите уравнения с дробями, которые требуют нескольких шагов

Вы можете быть ошеломлены, когда видите дроби в уравнении, поэтому мы собираемся показать метод решения уравнений с дробями, в котором вы используете общий знаменатель, чтобы исключить дроби из уравнения.Результатом этой операции будет новое уравнение, эквивалентное первому, но без дробей.

Обратите внимание на то, что каждый член в уравнении умножается на наименьший общий знаменатель. Вот что отличает его от оригинала!

ПРИМЕР

Решение: [латекс] \ frac {1} {8} x + \ frac {1} {2} = \ frac {1} {4} [/ latex].

Решение:

[латекс] \ frac {1} {8} x + \ frac {1} {2} = \ frac {1} {4} \ quad {LCD = 8} [/ latex]
Умножьте обе части уравнения на этот ЖК-дисплей, [латекс] 8 [/ латекс].Это очищает фракции. [латекс] \ color {красный} {8 (} \ frac {1} {8} x + \ frac {1} {2} \ color {red} {)} = \ color {red} {8 (} \ frac {1} {4} \ color {red} {)} [/ latex]
Используйте свойство распределения. [латекс] 8 \ cdot \ frac {1} {8} x + 8 \ cdot \ frac {1} {2} = 8 \ cdot \ frac {1} {4} [/ латекс]
Упростите — и заметьте, никаких дробей! [латекс] x + 4 = 2 [/ латекс]
Решите, используя общую стратегию решения линейных уравнений. [латекс] x + 4 \ color {red} {- 4} = 2 \ color {red} {- 4} [/ latex]
Упростить. [латекс] x = -2 [/ латекс]
Проверить: Пусть [латекс] x = -2 [/ латекс]

[латекс] \ frac {1} {8} x + \ frac {1} {2} = \ frac {1} {4} [/ latex]

[латекс] \ frac {1} {8} (\ color {red} {- 2}) + \ frac {1} {2} \ stackrel {\ text {?}} {=} \ Frac {1} { 4} [/ латекс]

[латекс] \ frac {-2} {8} + \ frac {1} {2} \ stackrel {\ text {?}} {=} \ Frac {1} {4} [/ latex]

[латекс] \ frac {-2} {8} + \ frac {4} {8} \ stackrel {\ text {?}} {=} \ Frac {1} {4} [/ latex]

[латекс] \ frac {2} {8} \ stackrel {\ text {?}} {=} \ Frac {1} {4} [/ latex]

[латекс] \ frac {1} {4} = \ frac {1} {4} \ quad \ checkmark [/ latex]

В последнем примере наименьший общий знаменатель был [латекс] 8 [/ латекс].Теперь ваша очередь найти ЖК-дисплей и очистить дроби, прежде чем решать эти линейные уравнения.

Обратите внимание, что после того, как мы очистили уравнение дробей, оно было похоже на те, которые мы решили ранее в этой главе. Мы изменили проблему на ту, которую уже знали, как решить!

Решите уравнения, очистив знаменатели

  1. Найдите наименьший общий знаменатель для всех дробей в уравнении.
  2. Умножьте обе части уравнения на этот ЖК-дисплей.Это очищает фракции.
  3. Выделите переменные члены с одной стороны и постоянные члены с другой стороны.
  4. Упростите обе стороны.
  5. Используйте свойство умножения или деления, чтобы коэффициент переменной был равен [latex] 1 [/ latex].

Вот пример с тремя переменными членами. После того, как вы очистите дроби с помощью ЖК-дисплея, вы упростите три члена переменных, а затем выделите переменную.

Пример

Решение: [латекс] 7 = \ frac {1} {2} x + \ frac {3} {4} x- \ frac {2} {3} x [/ latex].

Показать решение

Решение:
Мы хотим очистить дроби, умножив обе части уравнения на ЖК-дисплей всех дробей в уравнении.

Найдите наименьший общий знаменатель всех дробей в уравнении. [латекс] 7 = \ frac {1} {2} x + \ frac {3} {4} x- \ frac {2} {3} x \ quad {LCD = 12} [/ latex]
Умножьте обе части уравнения на [латекс] 12 [/ латекс]. [латекс] \ color {red} {12} (7) = \ color {red} {12} \ cdot (\ frac {1} {2} x + \ frac {3} {4} x- \ frac {2 } {3} x) [/ латекс]
Распространить. [латекс] 12 (7) = 12 \ cdot \ frac {1} {2} x + 12 \ cdot \ frac {3} {4} x-12 \ cdot \ frac {2} {3} x [/ латекс ]
Упростите — и заметьте, никаких дробей! [латекс] 84 = 6x + 9x-8x [/ латекс]
Объедините похожие термины. [латекс] 84 = 7x [/ латекс]
Разделить на [латекс] 7 [/ латекс]. [латекс] \ frac {84} {\ color {red} {7}} = \ frac {7x} {\ color {red} {7}} [/ latex]
Упростить. [латекс] 12 = x [/ латекс]
Проверить: Пусть [latex] x = 12 [/ latex].
[латекс] 7 = \ frac {1} {2} x + \ frac {3} {4} x- \ frac {2} {3} x [/ latex]

[латекс] 7 \ stackrel {\ text {?}} {=} \ Frac {1} {2} (\ color {red} {12}) + \ frac {3} {4} (\ color {red} {12}) — \ frac {2} {3} (\ color {red} {12}) [/ latex]

[латекс] 7 \ stackrel {\ text {?}} {=} 6 + 9-8 [/ латекс]

[латекс] 7 = 7 \ четырехугольник \ галочка [/ латекс]

А теперь попробуйте аналогичную задачу. Очистите дроби, упростите и решите.

Осторожно!

Одна из самых распространенных ошибок при очистке дробей — это забвение умножения ОБЕИХ сторон уравнения на ЖК-дисплей. Если ваш ответ не проходит, убедитесь, что вы умножили обе части уравнения на ЖК-дисплей.

В следующем примере у нас будут переменные и дроби с обеих сторон уравнения. После того, как вы очистите дроби с помощью ЖК-дисплея, вы увидите, что это уравнение похоже на уравнения с переменными с обеих сторон, которые мы решили ранее.Не забудьте выбрать переменную сторону и постоянную сторону, чтобы помочь вам организовать свою работу.

Пример

Решение: [латекс] x + \ frac {1} {3} = \ frac {1} {6} x- \ frac {1} {2} [/ latex].

Показать решение

Решение:

Найдите на ЖК-дисплее все дроби в уравнении. [латекс] x + \ frac {1} {3} = \ frac {1} {6} x- \ frac {1} {2}, \ quad {LCD = 6} [/ latex]
Умножьте обе стороны на ЖК-дисплей. [латекс] \ color {red} {6} (x + \ frac {1} {3}) = \ color {red} {6} (\ frac {1} {6} x- \ frac {1} {2 }) [/ латекс]
Распространить. [латекс] 6 \ cdot {x} +6 \ cdot \ frac {1} {3} = 6 \ cdot \ frac {1} {6} x-6 \ cdot \ frac {1} {2} [/ латекс ]
Упростите — больше никаких дробей! [латекс] 6x + 2 = x-3 [/ латекс]
Вычтите [латекс] x [/ латекс] с обеих сторон. [латекс] 6x- \ color {красный} {x} + 2 = x- \ color {красный} {x} -3 [/ latex]
Упростить. [латекс] 5x + 2 = -3 [/ латекс]
Вычтите 2 с обеих сторон. [латекс] 5x + 2 \ color {red} {- 2} = — 3 \ color {red} {- 2} [/ latex]
Упростить. [латекс] 5x = -5 [/ латекс]
Разделить на [латекс] 5 [/ латекс]. [латекс] \ frac {5x} {\ color {red} {5}} = \ frac {-5} {\ color {red} {5}} [/ latex]
Упростить. [латекс] x = -1 [/ латекс]
Проверить: Заменить [латекс] x = -1 [/ латекс].
[латекс] x + \ frac {1} {3} = \ frac {1} {6} x- \ frac {1} {2} [/ latex]

[латекс] (\ color {red} {- 1}) + \ frac {1} {3} \ stackrel {\ text {?}} {=} \ Frac {1} {6} (\ color {red} {-1}) — \ frac {1} {2} [/ latex]

[латекс] (- 1) + \ frac {1} {3} \ stackrel {\ text {?}} {=} — \ frac {1} {6} — \ frac {1} {2} [/ латекс ]

[латекс] — \ frac {3} {3} + \ frac {1} {3} \ stackrel {\ text {?}} {=} — \ frac {1} {6} — \ frac {3} { 6} [/ латекс]

[латекс] — \ frac {2} {3} \ stackrel {\ text {?}} {=} — \ frac {4} {6} [/ latex]

[латекс] — \ frac {2} {3} = — \ frac {2} {3} \ quad \ checkmark [/ latex]

Теперь вы можете попробовать решить уравнение с дробями, в котором переменные находятся по обе стороны от знака равенства.Ответ может быть дробным.

В следующем видео мы показываем еще один пример решения уравнения, которое содержит дроби и переменные по обе стороны от знака равенства.

В следующем примере мы начнем с уравнения, в котором переменный член заключен в скобки и умножен на дробь. Вы можете очистить дробь, или, если вы используете свойство распределения, оно удалит дробь. Вы понимаете почему?

ПРИМЕР

Решение: [латекс] 1 = \ frac {1} {2} \ left (4x + 2 \ right) [/ latex].

Показать решение

Решение:

[латекс] 1 = \ frac {1} {2} (4x + 2) [/ латекс]
Распространить. [латекс] 1 = \ frac {1} {2} \ cdot4x + \ frac {1} {2} \ cdot2 [/ latex]
Упростить. Теперь дробей нет! [латекс] 1 = 2x + 1 [/ латекс]
Вычтите 1 с обеих сторон. [латекс] 1 \ color {red} {- 1} = 2x + 1 \ color {red} {- 1} [/ latex]
Упростить. [латекс] 0 = 2x [/ латекс]
Разделить на [латекс] 2 [/ латекс]. [латекс] \ frac {0} {\ color {red} {2}} = \ frac {2x} {\ color {red} {2}} [/ latex]
Упростить. [латекс] 0 = x [/ латекс]
Проверить: Пусть [latex] x = 0 [/ latex].
[латекс] 1 = \ frac {1} {2} (4x + 2) [/ латекс]

[латекс] 1 \ stackrel {\ text {?}} {=} \ Frac {1} {2} (4 (\ color {red} {0}) + 2) [/ latex]

[латекс] 1 \ stackrel {\ text {?}} {=} \ Frac {1} {2} (2) [/ латекс]

[латекс] 1 \ stackrel {\ text {?}} {=} \ Frac {2} {2} [/ latex]

[латекс] 1 = 1 \ четырехугольник \ галочка [/ латекс]

Теперь вы можете попробовать решить уравнение, в котором переменный член в скобках умножен на дробь.

4.9: Решение уравнений с дробями

Отмена вычитания

Мы все еще можем добавить одинаковую сумму к обеим частям уравнения, не меняя решения.

Пример 1

Решите относительно x : \ (x — \ frac {5} {6} = \ frac {1} {3} \).

Раствор

Чтобы «отменить» вычитание 5/6, прибавьте 5/6 к обеим сторонам уравнения и упростите.

\ [\ begin {align} x — \ frac {5} {6} = \ frac {1} {3} ~ & \ textcolor {red} {\ text {Исходное уравнение.}} \\ x — \ frac {5} {6} + \ frac {5} {6} = \ frac {1} {3} + \ frac {5} {6} ~ & \ textcolor {red} {\ текст {Add} \ frac {5} {6} \ text {в обе стороны.}} \\ x = \ frac {1 \ cdot 2} {3 \ cdot 2} + \ frac {5} {6} ~ & \ textcolor {red} {\ text {Эквивалентные дроби, LCD = 6.}} \\ x = \ frac {2} {6} + \ frac {5} {6} ~ & \ textcolor {red} {\ text { Упростить.}} \\ x = \ frac {7} {6} ~ & \ textcolor {red} {\ text {Add.}} \ End {align} \ nonumber \]

Вполне допустимо оставлять свой ответ в виде неправильной дроби. Если вы хотите или если вас попросят сделать это, вы можете изменить свой ответ на смешанную дробь (7, разделенное на 6, будет равно 1, а остаток — 1).То есть \ (x = 1 \ frac {1} {6} \).

Проверка решения

Замените 7/6 на x в исходном уравнении и упростите.

\ [\ begin {align} x — \ frac {5} {6} = \ frac {1} {3} ~ & \ textcolor {red} {\ text {Исходное уравнение.}} \\ \ frac {7} {6} — \ frac {5} {6} = \ frac {1} {3} ~ & \ textcolor {red} {\ text {Заменить 7/6} x.} \\ \ frac {2} {6 } = \ frac {1} {3} ~ & \ textcolor {red} {\ text {Subtract.}} \\ \ frac {1} {3} = \ frac {1} {3} ~ & \ textcolor {красный } {\ text {Уменьшить.}} \ конец {выровнено} \ nonumber \]

Поскольку последнее утверждение верно, мы заключаем, что 7/6 является решением уравнения x — 5/6 = 1/3.

Отмена добавления

Вы по-прежнему можете вычесть одинаковую сумму из обеих частей уравнения, не меняя решение.

Пример 2

Решите относительно x : \ (x + \ frac {2} {3} = — \ frac {3} {5} \).

Раствор

Чтобы «отменить» сложение 2/3, вычтите 2/3 из обеих частей уравнения и упростите.

\ [\ begin {align} x + \ frac {2} {3} = — \ frac {3} {5} ~ & \ textcolor {red} {\ text {Исходное уравнение.}} \\ x + \ frac {2} {3} — \ frac {2} {3} = — \ frac {3} {5} — \ frac {2} {3} ~ & \ textcolor {red} {\ text {Subtract} \ frac { 2} {3} \ text {с обеих сторон.}} \\ x = — \ frac {3 \ cdot 3} {5 \ cdot 3} — \ frac {2 \ cdot 5} {3 \ cdot 5} ~ & \ textcolor {red} {\ text {Эквивалентные дроби, LCD = 15.}} \\ x = — \ frac {9} {15} — \ frac {10} {15} ~ & \ textcolor {red} {\ text {Упростить.}} \\ x = — \ frac {19} {15} ~ & \ textcolor {red} {\ text {Вычесть.}} \ конец {выровнено} \ nonumber \]

Читателям предлагается проверить это решение в исходном уравнении.

Упражнение

Решите относительно x : \ (x + \ frac {3} {4} = — \ frac {1} {2} \)

Ответ

−5/4

Отмена умножения

Мы «отменяем» умножение делением. Например, чтобы решить уравнение 2 x = 6, мы разделим обе части уравнения на 2.Аналогичным образом мы могли бы разделить обе части уравнения

\ [\ frac {3} {5} x = \ frac {4} {10} \ nonumber \]

на 3/5. Однако более эффективно использовать обратные. Для удобства мы напоминаем читателям о мультипликативном обратном свойстве .

Мультипликативное обратное свойство

Пусть a / b будет произвольной дробью. Число b / a называется обратным мультипликативным числом или обратным числом a / b .Произведение обратных величин 1.

\ [\ frac {a} {b} \ cdot \ frac {b} {a} = 1. \ nonumber \]

Давайте применим наши знания о взаимных вычислениях.

Пример 3

Решите относительно x : \ (\ frac {3} {5} x = \ frac {4} {10} \).

Раствор

Чтобы «отменить» умножение на 3/5, умножьте обе части на обратное 5/3 и упростите.

\ [\ begin {align} \ frac {3} {5} x = \ frac {4} {10} ~ & \ textcolor {red} {\ text {Исходное уравнение.}} \\ \ frac {5} {3} \ left (\ frac {3} {5} x \ right) = \ frac {5} {3} \ left (\ frac {4} {10} \ right) & ~ \ textcolor {red} {\ text {Умножьте обе стороны на 5/3.}} \\ \ left (\ frac {5} {3} \ cdot \ frac {3} {5} \ right) x = \ frac {20} {30} ~ & \ textcolor {red} {\ begin {array} {l} \ text {Слева используйте ассоциативное свойство для перегруппировки.} \\ \ text {Справа — умножение.} \ end {array}} \\ 1x = \ frac {2} {3} ~ & \ textcolor {red} {\ begin {array} {l} \ text {Слева} \ frac {5} {3} \ cdot \ frac {3} {5} = 1. \\ \ text {Справа уменьшите:} \ frac {20} {30} = \ frac {2} {3}.\ end {array}} \\ x = \ frac {2} {3} ~ & \ textcolor {red} {\ text {Слева} 1x = x.} \ end {align} \ nonumber \]

Проверка решения

Замените 2/3 на x в исходном уравнении и упростите.

\ [\ begin {align} \ frac {3} {5} x = \ frac {4} {10} ~ & \ textcolor {red} {\ text {Исходное уравнение.}} \\ \ frac {3} { 5} \ left (\ frac {2} {3} \ right) = \ frac {4} {10} ~ & \ textcolor {red} {\ text {Замените 2/3 на} x.} \\ \ frac { 6} {15} = \ frac {4} {10} ~ & \ textcolor {red} {\ text {Умножение числителей; умножьте знаменатели.}} \\ \ frac {2} {5} = \ frac {2} {5} ~ & \ textcolor {red} {\ text {Уменьшить обе стороны до наименьших значений.}} \ end {align} \ nonumber \]

Поскольку это последнее утверждение верно, мы заключаем, что 2/3 является решением уравнения (3/5) x = 4/10.

Упражнение

Решите относительно y : \ (\ frac {2} {3} y = \ frac {4} {5} \)

Ответ

6/5

Пример 4

Решите относительно x : \ (- \ frac {8} {9} x = \ frac {5} {18} \).

Раствор

Чтобы «отменить» умножение на −8/9, умножьте обе части на обратное −9/8 и упростите.

\ [\ begin {align} — \ frac {8} {9} x = \ frac {5} {18} ~ & \ textcolor {red} {\ text {Исходное уравнение.}} \\ — \ frac {9 } {8} \ left (- \ frac {8} {9} x \ right) = — \ frac {9} {8} \ left (\ frac {5} {18} \ right) ~ & \ textcolor {красный } {\ text {Умножьте обе стороны на} -9/8.} \\ \ left [- \ frac {9} {8} \ cdot \ left (- \ frac {8} {9} \ right) \ right] x = — \ frac {3 \ cdot 3} {2 \ cdot 2 \ cdot 2} \ cdot \ frac {5} {2 \ cdot 3 \ cdot 3} ~ & \ textcolor {red} {\ begin {array} { l} \ text {Слева используйте ассоциативное свойство для перегруппировки.} \\ \ text {Справа, простой множитель.} \ end {array}} 1x = \ frac {\ cancel {3} \ cdot \ cancel {3}} {2 \ cdot 2 \ cdot 2} \ cdot \ frac {5} {2 \ cdot \ cancel {3} \ cdot \ cancel {3}} ~ & \ textcolor {red} {\ begin {array} {l} \ text {Слева} — \ frac {9 } {8} \ cdot \ left (- \ frac {8} {9} \ right) = 1. \\ \ text {Справа отмените общие множители.} \ End {array}} \\ x = — \ frac {5} {16} ~ & \ textcolor {red} {\ text {Слева} 1x = x. \ text {Умножение справа.}} \ end {Выровнено} \ nonumber \]

Читателям предлагается проверить это решение в исходном уравнении.

Упражнение

Решите относительно z: \ (- \ frac {2} {7} z = \ frac {4} {21} \)

Ответ

-2/3

Удаление дробей из уравнения

Хотя техника, продемонстрированная в предыдущих примерах, является надежной математической техникой, работа с дробями в уравнении не всегда является наиболее эффективным использованием вашего времени.

Удаление дробей из уравнения

Чтобы удалить все дроби из уравнения, умножьте обе части уравнения на наименьший общий знаменатель дробей, которые встречаются в уравнении.

Давайте реализуем эту идею.

Пример 5

В примере 1 нас попросили решить следующее уравнение для x :

\ [x — \ frac {5} {6} = \ frac {1} {3}. \ Nonumber \]

Найдите минутку, чтобы рассмотреть технику решения в примере 1. Теперь мы решим это уравнение, сначала удалив все дроби из уравнения.

Раствор

Умножьте обе части уравнения на наименьший общий знаменатель дробей, фигурирующих в уравнении.

\ [\ begin {align} x — \ frac {5} {6} = \ frac {1} {3} ~ & \ textcolor {red} {\ text {Исходное уравнение.}} \\ 6 \ left (x — \ frac {5} {6} \ right) = 6 \ left (\ frac {1} {3} \ right) ~ & \ textcolor {red} {\ text {Умножьте обе стороны на 6.}} \\ 6x — 6 \ left (\ frac {5} {6} \ right) = 6 \ left (\ frac {1} {3} \ right) ~ & \ textcolor {red} {\ text {Распределить 6.}} \ \ 6x-5 = 2 ~ & \ textcolor {red} {\ text {Сначала умножьте с каждой стороны.}} \\ ~ & \ textcolor {red} {6 \ left (\ frac {5} {6} \ right ) = 5 \ text {и} 6 \ left (\ frac {1} {3} \ right) = 2.} \ конец {выровнено} \ nonumber \]

Обратите внимание, что уравнение теперь полностью очищено от дробей, что значительно упрощает его решение.

\ [\ begin {align} 6x — 5 + 5 = 2 + 5 ~ & \ textcolor {red} {\ text {Добавьте 5 с обеих сторон.}} \\ 6x = 7 ~ & \ textcolor {red} {\ text {Упростите обе стороны.}} \\ \ frac {6x} {6} = \ frac {7} {6} ~ & \ textcolor {red} {\ text {Разделите обе стороны на 6.}} \\ x = \ frac {7} {6} ~ & \ textcolor {red} {\ text {Упростить.}} \ end {align} \ nonumber \]

Обратите внимание, что это то же самое решение, что и в Примере 1.

Упражнение

Решить относительно t : \ (t — \ frac {2} {7} = — \ frac {1} {4} \)

Ответ

1/28

Пример 6

В примере 4 нас попросили решить следующее уравнение для x .

\ [- \ frac {8} {9} x = \ frac {5} {18} \ nonumber \]

Найдите минутку, чтобы просмотреть решение в примере 4. Теперь мы решим это уравнение, сначала удалив все дроби из уравнения.

Раствор

Умножьте обе части уравнения на наименьший общий знаменатель дробей, фигурирующих в уравнении.

\ [\ begin {align} — \ frac {8} {9} x = \ frac {5} {18} ~ & \ textcolor {red} {\ text {Исходное уравнение.}} \\ 18 \ left (- \ frac {8} {9} x \ right) = 18 \ left (\ frac {5} {18} \ right) ~ & \ textcolor {red} {\ text {Умножаем обе стороны на 18.}} \\ — 16x = 5 ~ & \ textcolor {red} {\ text {С каждой стороны, отменить и умножить.}} \\ ~ & \ textcolor {red} {18 \ left (- \ frac {8} {9} \ right) = -16 \ text {и} 18 \ left (\ frac {5} {18} \ right) = 5.} \ конец {выровнено} \ nonumber \]

Обратите внимание, что уравнение теперь полностью избавлено от дробей. Продолжая,

\ [\ begin {align} \ frac {-16x} {- 16} = \ frac {5} {- 16} ~ & \ textcolor {red} {\ text {Разделите обе стороны на} -16.} \\ x = — \ frac {5} {16} ~ & \ textcolor {red} {\ text {Упростить.}} \ end {align} \ nonumber \]

Обратите внимание, что это то же самое решение, что и в примере 4.

Упражнение

Решить относительно u :

\ [- \ frac {7} {9} u = \ frac {14} {27} \ nonumber \]

Ответ

-2/3

Пример 7

Решите относительно x : \ (\ frac {2} {3} x + \ frac {3} {4} = \ frac {1} {2} \).

Раствор

Умножьте обе части уравнения на наименьший общий знаменатель дробей, фигурирующих в уравнении.

\ [\ begin {align} \ frac {2} {3} x + \ frac {3} {4} = \ frac {1} {2} ~ & \ textcolor {red} {\ text {Исходное уравнение.} } \\ 12 \ left (\ frac {2} {3} x + \ frac {3} {4} = \ right) = 12 \ left (\ frac {1} {2} \ right) ~ & \ textcolor { red} {\ text {Умножьте обе стороны на 12.}} \\ 12 \ left (\ frac {2} {3} x \ right) + 12 \ left (\ frac {3} {4} \ right) = 12 \ left (\ frac {1} {2} \ right) ~ & \ textcolor {red} {\ text {Слева распределите 12 штук.}} \\ 8x + 9 = 6 ~ & \ textcolor {red} {\ text {Умножение:} 12 \ left (\ frac {2} {3} x \ right) = 8x, ~ 12 \ left (\ frac { 3} {4} \ right) = 9,} \\ ~ & \ textcolor {red} {\ text {и} 12 \ left (\ frac {1} {2} \ right) = 6.} \ end {выровнено } \ nonumber \]

Обратите внимание, что уравнение теперь полностью избавлено от дробей. Нам нужно изолировать члены, содержащие x на одной стороне уравнения.

\ [\ begin {align} 8x + 9 — 9 = 6 — 9 ~ & \ textcolor {red} {\ text {Вычтите 9 с обеих сторон.}} \\ 8x = — 3 ~ & \ textcolor {red} {\ text {Упростите обе стороны.}} \\ \ frac {8x} {8} = \ frac {-3} {8} ~ & \ textcolor { red} {\ text {Разделите обе стороны на 8.}} \\ x = — \ frac {3} {8} ~ & \ textcolor {red} {\ text {Упростите обе стороны.}} \ end {align} \ nonumber \]

Читателям предлагается проверить это решение в исходном уравнении.

Упражнение

Решите относительно r : \ (\ frac {3} {4} r + \ frac {2} {3} = \ frac {1} {2} \)

Ответ

-2/9

Пример 8

Решите относительно x : \ (\ frac {2} {3} — \ frac {3x} {4} = \ frac {x} {2} — \ frac {1} {8}.\)

Раствор

Умножьте обе части уравнения на наименьший общий знаменатель дробей в уравнении.

\ [\ begin {align} \ frac {2} {3} — \ frac {3x} {4} = \ frac {x} {2} — \ frac {1} {8} ~ & \ textcolor {красный} {\ text {Исходное уравнение.}} \\ 24 \ left (\ frac {2} {3} — \ frac {3x} {4} \ right) = 24 \ left (\ frac {x} {2} — \ frac {1} {8} \ right) ~ & \ textcolor {red} {\ text {Умножьте обе стороны на 24.}} \\ 24 \ left (\ frac {2} {3} \ right) — 24 \ left (\ frac {3x} {4} \ right) = 24 \ left (\ frac {x} {2} \ right) — 24 \ left (\ frac {1} {8} \ right) ~ & \ textcolor {красный } {\ text {С обеих сторон распределите по 24 штуки.}} \\ 16 — 18x = 12x — 3 ~ & \ textcolor {red} {\ text {Left:} 24 \ left (\ frac {2} {3} \ right) = 16, ~ 24 \ left (\ frac {3x} {4} \ right) = 18x.} \\ ~ & \ textcolor {red} {\ text {Right:} 24 \ left (\ frac {x} {2} \ right) = 12x, ~ 24 \ слева (\ frac {1} {8} \ right) = 3.} \ end {выравнивается} \ nonumber \]

Обратите внимание, что уравнение теперь полностью избавлено от дробей. Нам нужно изолировать члены, содержащие x, на одной стороне уравнения.

\ [\ begin {align} 16 — 18x — 12x = 12x — 3 — 12x ~ & \ textcolor {red} {\ text {Subtract} 12x \ text {с обеих сторон.}} \\ 16 — 30x = -3 ~ & \ textcolor {красный} {\ begin {выровненный} \ text {Left:} -18x — 12x = -30x. \\ \ text {Right:} 12x — 12x = 0. \ end {align}} \\ 16 — 30x — 16 = -3 — 16 ~ & \ textcolor {red} {\ text {Вычтите 16 с обеих сторон.} } \\ -30x = -19 ~ & \ textcolor {красный} {\ begin {выровненный} \ text {Left:} 16-16 = 0. \\ \ text {Right:} -3 — 16 = -19. \ end {align}} \\ \ frac {-30x} {- 30} = \ frac {-19} {- 30} ~ & \ textcolor {red} {\ text {Разделите обе стороны на} -30.} \ \ x = \ frac {19} {30} ~ & \ textcolor {red} {\ text {Упростите обе стороны.}} \ конец {выровнено} \ nonumber \]

Читателям предлагается проверить это решение в исходном уравнении.

Упражнение

Решить относительно с : \ (\ frac {3} {2} — \ frac {2s} {5} = \ frac {s} {3} — \ frac {1} {5} \).

Ответ

Добавьте сюда тексты. Не удаляйте сначала этот текст.

Приложения

Давайте посмотрим на некоторые приложения, в которых используются уравнения, содержащие дроби.Для удобства мы повторяем требования для решения проблем Word .

Требования к решению проблем Word

  1. Настройка словаря переменных . Вы должны сообщить своим читателям, что представляет собой каждая переменная в вашей проблеме. Этого можно добиться несколькими способами:
    1. Такие утверждения, как «Пусть P представляет периметр прямоугольника».
    2. Пометка неизвестных значений переменными в таблице.
    3. Обозначение неизвестных величин на эскизе или диаграмме.
  2. Установите уравнение . Каждое решение проблемы со словом должно включать тщательно составленное уравнение, которое точно описывает ограничения в постановке задачи.
  3. Решите уравнение . Вы всегда должны решать уравнение, заданное на предыдущем шаге.
  4. Ответьте на вопрос . Этот шаг легко упустить из виду. Например, в задаче может задаваться вопрос о возрасте Джейн, но решение вашего уравнения дает возраст сестры Джейн, Лиз.Убедитесь, что вы ответили на исходный вопрос, заданный в задаче. Ваше решение должно быть записано в предложении с соответствующими единицами. 5. Оглянитесь назад. Важно отметить, что этот шаг не означает, что вы должны просто проверить свое решение в своем уравнении. В конце концов, возможно, что ваше уравнение неверно моделирует ситуацию проблемы, поэтому у вас может быть действительное решение неправильного уравнения. Важный вопрос: «Имеет ли ваш ответ смысл на основе слов в исходной постановке задачи.”

Пример 9

В третьей четверти баскетбольного матча дикторы сообщили толпе, что на игру пришло 12 250 человек. Если это две трети вместимости, найдите полную вместимость баскетбольной арены.

Раствор

Мы соблюдаем требования для решения проблем Word .

1. Настройка словаря переменных . Пусть F представляет полную пассажировместимость. Примечание: гораздо лучше использовать переменную, которая «звучит как» величина, которую она представляет.В этом случае использование F для представления полной вместимости пассажиров более наглядно, чем использование x для представления полной вместимости.

2. Установите уравнение . Две трети от полной вместимости составляет 12 250 человек.

\ [\ begin {align} \ colorbox {cyan} {Две трети} & \ text {of} & \ colorbox {cyan} {Полная вместимость} & \ text {is} & 12,250 \\ \ frac {2} {3} & \ cdot & F & = & 12,250 \ end {align} \ nonumber \]

Следовательно, уравнение

\ [\ frac {2} {3} F = 12250.\ nonumber \]

3. Решите уравнение . Умножьте обе части на 3, чтобы очистить дроби, затем решите.

\ [\ begin {align} \ frac {2} {3} F = 12250 ~ & \ textcolor {red} {\ text {Исходное уравнение.}} \\ 3 \ left (\ frac {2} {3} F \ right) = 3 (12250) ~ & \ textcolor {red} {\ text {Умножьте обе стороны на 3.}} \\ 2F = 36750 ~ & \ textcolor {red} {\ text {Упростите обе стороны.}} \ \ \ frac {2F} {2} = \ frac {36750} {2} ~ & \ textcolor {red} {\ text {Разделите обе стороны на 2.}} \\ F = 18375 ~ & \ textcolor {red} { \ text {Упростите обе стороны.}} \ конец {выровнено} \ nonumber \]

4. Ответьте на вопрос . Полная вместимость — 18 375 человек.

5. Оглянись назад . В словах проблемы указано, что 2/3 пассажировместимости составляет 12 250 человек. Давайте возьмем две трети нашего ответа и посмотрим, что мы получим.

\ [\ begin {align} \ frac {2} {3} \ cdot 18375 & = \ frac {2} {3} \ cdot \ frac {18375} {1} \\ & = \ frac {2} {3 } \ cdot \ frac {3 \ cdot 6125} {1} \\ & = \ frac {2} {\ cancel {3}} \ cdot \ frac {\ cancel {3} \ cdot 6125} {1} \\ & = 12250 \ конец {выровнено} \ nonumber \]

Это правильная посещаемость, поэтому наше решение правильное.

Упражнение

Посещаемость игры «Селтикс» составила 9 510 человек. Если это 3/4 вместимости, какова вместимость арены «Селтикс»?

Ответ

12 680

Пример 10

Площадь треугольника составляет 20 квадратных дюймов. Если длина основания составляет \ (2 \ frac {1} {2} \) дюймов, найдите высоту (высоту) треугольника.

Раствор

Мы соблюдаем требования для решения проблем Word .

1. Настройка словаря переменных . Наш словарь переменных будет иметь форму хорошо размеченной диаграммы.

2. Установите уравнение . Площадь A треугольника с основанием b и высотой h равна

.

\ [A = \ frac {1} {2} bh. \ Nonumber \]

Заменить A = 20 и b = \ (2 \ frac {1} {2} \).

\ [20 = \ frac {1} {2} \ left (2 \ frac {1} {2} \ right) h. \ Nonumber \]

3. Решите уравнение . Измените смешанную дробь на неправильную дробь, а затем упростите.

\ [\ begin {align} 20 = \ frac {1} {2} \ left (2 \ frac {1} {2} \ right) h ~ & \ textcolor {red} {\ text {Исходное уравнение.}} \\ 20 = \ frac {1} {2} \ left (\ frac {5} {2} \ right) h ~ & \ textcolor {red} {\ text {Смешано с неправильным:} 2 \ frac {1} { 2} = \ frac {5} {2}.} \\ 20 = \ left (\ frac {1} {2} \ cdot \ frac {5} {2} \ right) h ~ & \ textcolor {red} { \ text {Ассоциативное свойство.}} \\ 20 = \ frac {5} {4} h ~ & \ textcolor {red} {\ text {Умножение:} \ frac {1} {2} \ cdot \ frac {5} {2} = \ frac {5} {4}.} \ конец {выровнено} \ nonumber \]

Теперь умножьте обе части на 4/5 и решите.

\ [\ begin {align} \ frac {4} {5} (20) = \ frac {4} {5} \ left (\ frac {5} {4} h \ right) ~ & \ textcolor {красный} {\ text {Умножьте обе стороны на 4/5.}} \\ 16 = h ~ & \ textcolor {red} {\ text {Simplify:} \ frac {4} {5} (20) = 16} \\ ~ & \ textcolor {красный} {\ text {и} \ frac {4} {5} \ cdot \ frac {5} {4} = 1.} \ end {align} \ nonumber \]

4. Ответьте на вопрос . Высота треугольника 16 дюймов.

5. Оглянись назад . Если высота составляет 16 дюймов, а основание — \ (2 \ frac {1} {2} \) дюймов, то площадь равна

.

\ [\ begin {align} A & = \ frac {1} {2} \ left (2 \ frac {1} {2} \ right) (16) \\ & = \ frac {1} {2} \ cdot \ frac {5} {2} \ cdot \ frac {16} {1} \\ & = \ frac {5 \ cdot 16} {2 \ cdot 2} \\ & = \ frac {(5) \ cdot ( 2 \ cdot 2 \ cdot 2 \ cdot 2)} {(2) \ cdot (2)} \\ & = \ frac {5 \ cdot \ cancel {2} \ cdot \ cancel {2} \ cdot 2 \ cot 2 } {\ cancel {2} \ cdot \ cancel {2}} & = 20 \ end {align} \ nonumber \]

Это правильная площадь (20 квадратных дюймов), поэтому наше решение правильное.

Упражнение

Площадь треугольника составляет 161 квадратный фут. Если основание треугольника имеет размер \ (40 \ frac {1} {4} \) футов, найдите высоту треугольника.

Ответ

8 футов

Упражнения

1. Является ли 1/4 решением уравнения \ (x + \ frac {5} {8} = \ frac {5} {8} \)?

2. Является ли 1/4 решением уравнения \ (x + \ frac {1} {3} = \ frac {5} {12} \)?

3. Является ли −8/15 решением уравнения \ (\ frac {1} {4} x = — \ frac {1} {15} \)?

4.Является ли −18/7 решением уравнения \ (- \ frac {3} {8} x = \ frac {25} {28} \)?

5. Является ли 1/2 решением уравнения \ (x + \ frac {4} {9} = \ frac {17} {18} \)?

6. Является ли 1/3 решением уравнения \ (x + \ frac {3} {4} = \ frac {13} {12} \)?

7. Является ли 3/8 решением уравнения \ (x — \ frac {5} {9} = — \ frac {13} {72} \)?

8. Является ли 1/2 решением уравнения \ (x — \ frac {3} {5} = — \ frac {1} {10} \)?

9. Является ли 2/7 решением уравнения \ (x — \ frac {4} {9} = — \ frac {8} {63} \)?

10.Является ли 1/9 решением уравнения \ (x — \ frac {4} {7} = — \ frac {31} {63} \)?

11. Является ли 8/5 решением уравнения \ (\ frac {11} {14} x = \ frac {44} {35} \)?

12. Является ли 16/9 решением уравнения \ (\ frac {13} {18} x = \ frac {104} {81} \)?


В упражнениях 13-24 решите уравнение и упростите свой ответ.

13. \ (2x — 3 = 6x + 7 \)

14. \ (9x — 8 = −9x — 3 \)

15. \ (- 7x + 4 = 3x \)

16. \ (6x + 9 = −6x \)

17.\ (- 2x = 9x — 4 \)

18. \ (- 6x = −9x + 8 \)

19. \ (- 8x = 7x — 7 \)

20. \ (- 6x = 5x + 4 \)

21. \ (- 7x + 8 = 2x \)

22. \ (- х — 7 = 3х \)

23. \ (- 9x + 4 = 4x — 6 \)

24. \ (- 2x + 4 = x — 7 \)


В упражнениях 25–48 решите уравнение и упростите свой ответ.

25. \ (x + \ frac {3} {2 = \ frac {1} {2} \)

26. \ (x — \ frac {3} {4} = \ frac {1} {4} \)

27. \ (- \ frac {9} {5} x = \ frac {1} {2} \)

28.\ (\ frac {7} {3} x = — \ frac {7} {2} \)

29. \ (\ frac {3} {8} x = \ frac {8} {7} \)

30. \ (- \ frac {1} {9} x = — \ frac {3} {5} \)

31. \ (\ frac {2} {5} x = — \ frac {1} {6} \)

32. \ (\ frac {1} {6} x = \ frac {2} {9} \)

33. \ (- \ frac {3} {2} x = \ frac {8} {7} \)

34. \ (- \ frac {3} {2} x = — \ frac {7} {5} \)

35. \ (x + \ frac {3} {4} = \ frac {5} {9} \)

36. \ (x — \ frac {1} {9} = — \ frac {3} {2} \)

37. \ (x — \ frac {4} {7} = \ frac {7} {8} \)

38.\ (x + \ frac {4} {9} = — \ frac {3} {4} \)

39. \ (x + \ frac {8} {9} = \ frac {2} {3} \)

40. \ (x — \ frac {5} {6} = \ frac {1} {4} \)

41. \ (x + \ frac {5} {2} = — \ frac {9} {8} \)

42. \ (x + \ frac {1} {2} = \ frac {5} {3} \)

43. \ (- \ frac {8} {5} x = \ frac {7} {9} \)

44. \ (- \ frac {3} {2} x = — \ frac {5} {9} \)

45. \ (x — \ frac {1} {4} = — \ frac {1} {8} \)

46. \ (x — \ frac {9} {2} = — \ frac {7} {2} \)

47. \ (- \ frac {1} {4} x = \ frac {1} {2} \)

48.\ (- \ frac {8} {9} x = — \ frac {8} {3} \)


В упражнениях 49–72 решите уравнение и упростите свой ответ.

49. \ (- \ frac {7} {3} x — \ frac {2} {3} = \ frac {3} {4} x + \ frac {2} {3} \)

50. \ (\ frac {1} {2} x — \ frac {1} {2} = \ frac {3} {2} x + \ frac {3} {4} \)

51. \ (- \ frac {7} {2} x — \ frac {5} {4} = \ frac {4} {5} \)

52. \ (- \ frac {7} {6} x + \ frac {5} {6} = — \ frac {8} {9} \)

53. \ (- \ frac {9} {7} x + \ frac {9} {2} = — \ frac {5} {2} \)

54.\ (\ frac {5} {9} x — \ frac {7} {2} = \ frac {1} {4} \)

55. \ (\ frac {1} {4} x — \ frac {4} {3} = — \ frac {2} {3} \)

56. \ (\ frac {8} {7} x + \ frac {3} {7} = \ frac {5} {3} \)

57. \ (\ frac {5} {3} x + \ frac {3} {2} = — \ frac {1} {4} \)

58. \ (\ frac {1} {2} x — \ frac {8} {3} = — \ frac {2} {5} \)

59. \ (- \ frac {1} {3} x + \ frac {4} {5} = — \ frac {9} {5} x — \ frac {5} {6} \)

60. \ (- \ frac {2} {9} x — \ frac {3} {5} = \ frac {4} {5} x — \ frac {3} {2} \)

61. \ (- \ frac {4} {9} x — \ frac {8} {9} = \ frac {1} {2} x — \ frac {1} {2} \)

62.\ (- \ frac {5} {4} x — \ frac {5} {3} = \ frac {8} {7} x + \ frac {7} {3} \)

63. \ (\ frac {1} {2} x — \ frac {1} {8} = — \ frac {1} {8} x + \ frac {5} {7} \)

64. \ (- \ frac {3} {2} x + \ frac {8} {3} = \ frac {7} {9} x — \ frac {1} {2} \)

65. \ (- \ frac {3} {7} x — \ frac {1} {3} = — \ frac {1} {9} \)

66. \ (\ frac {2} {3} x + \ frac {2} {9} = — \ frac {9} {5} \)

67. \ (- \ frac {3} {4} x + \ frac {2} {7} = \ frac {8} {7} x — \ frac {1} {3} \)

68. \ (\ frac {1} {2} x + \ frac {1} {3} = — \ frac {5} {2} x — \ frac {1} {4} \)

69.\ (- \ frac {3} {4} x — \ frac {2} {3} = — \ frac {2} {3} x — \ frac {1} {2} \)

70. \ (\ frac {1} {3} x — \ frac {5} {7} = \ frac {3} {2} x + \ frac {4} {3} \)

71. \ (- \ frac {5} {2} x + \ frac {9} {5} = \ frac {5} {8} \)

72. \ (\ frac {9} {4} x + \ frac {4} {3} = — \ frac {1} {6} \)


73. На местном футбольном матче дикторы сообщили толпе, что на игру пришло 4 302 человека. Если это 2/9 вместимости, найдите полную вместимость футбольного стадиона.

74.На местном баскетбольном матче дикторы сообщили толпе, что на игру пришло 5 394 человека. Если это 2/7 вместимости, найдите полную вместимость баскетбольного стадиона.

75. Площадь треугольника составляет 51 квадратный дюйм. Если длина основания составляет \ (8 \ frac {1} {2} \) дюймов, найдите высоту (высоту) треугольника.

76. Площадь треугольника составляет 20 квадратных дюймов. Если длина основания составляет \ (2 \ frac {1} {2} \) дюймов, найдите высоту (высоту) треугольника.

77. Площадь треугольника составляет 18 квадратных дюймов. Если длина основания составляет \ (4 \ frac {1} {2} \) дюймов, найдите высоту (высоту) треугольника.

78. Площадь треугольника составляет 44 квадратных дюйма. Если длина основания составляет \ (5 \ frac {1} {2} \) дюймов, найдите высоту (высоту) треугольника.

79. На местном хоккейном матче дикторы сообщили толпе, что на игру пришло 4 536 человек. Если это 2/11 вместимости, найдите полную вместимость хоккейного стадиона.

80. На местном футбольном матче дикторы сообщили толпе, что на игру пришло 6 970 человек. Если это 2/7 вместимости, найдите полную вместимость футбольного стадиона.


81. Пираты . Около одной трети пиратских нападений в мире в 2008 году произошло у побережья Сомали. Если было 111 пиратских нападений у побережья Сомали, оцените количество пиратских нападений во всем мире в 2008 году.

82. Ядерный арсенал . U.С. и Россия договорились сократить ядерные арсеналы ядерного оружия большой дальности примерно на треть, до 1 550. Сколько сейчас ядерного оружия большой дальности? Associated Press-Times-Standard 04/04/10 Ядерный центр обеспокоен сокращением ракет.

83. Хранилище семян . В Глобальном хранилище семян на Свальбарде собрано полмиллиона образцов семян, и теперь в нем хранится не менее одной трети семян сельскохозяйственных культур в мире. Оцените общее количество семян сельскохозяйственных культур в мире. Associated Press-Times-Standard 15.03.10 Норвегия в хранилище семян судного дня достигла отметки в полмиллиона.

84. Товарный поезд . Поезд «Юнион Пасифик» длиной в три с половиной мили примерно в 2 1 2 раза длиннее обычного грузового поезда. Какова длина обычного грузового поезда? Associated Press-Times-Standard 13.01.10 Необычно длинный поезд вызывает опасения по поводу безопасности.


Решайте уравнения с переменными и константами с обеих сторон — предалгебра

Цели обучения

К концу этого раздела вы сможете:

  • Решите уравнение с константами с обеих сторон
  • Решите уравнение с переменными с обеих сторон
  • Решите уравнение с переменными и константами с обеих сторон
  • Решите уравнения, используя общую стратегию

Перед тем, как начать, пройдите тест на готовность.

  1. Simplify:
    Если вы пропустили эту проблему, просмотрите (рисунок).
  2. Решить:
    Если вы пропустили эту проблему, просмотрите (рисунок).
  3. Решить:
    Если вы пропустили эту проблему, просмотрите (рисунок).

Решите уравнение с константами с обеих сторон

Возможно, вы заметили, что во всех уравнениях, которые мы решили до сих пор, все переменные члены находились только на одной стороне уравнения, а константы — на другой. Это происходит не все время, поэтому теперь мы увидим, как решать уравнения, в которых переменные и / или постоянные члены находятся по обе стороны уравнения.

Наша стратегия будет включать выбор одной стороны уравнения в качестве переменной, а другой стороны уравнения в качестве постоянной. Затем мы будем использовать свойства равенства вычитания и сложения, шаг за шагом, чтобы собрать все переменные члены вместе на одной стороне уравнения и постоянные члены вместе на другой стороне.

Сделав это, мы преобразуем уравнение, которое начиналось с переменных и констант с обеих сторон, в форму. Мы уже знаем, как решать уравнения этой формы, используя свойства равенства или деления или умножения.

Решить:

Решение

В этом уравнении переменная находится только в левой части. Левую часть имеет смысл называть стороной переменных. Следовательно, правая сторона будет постоянной стороной. Мы напишем метки над уравнением, чтобы помочь нам запомнить, что куда идет.

Решить:

Решить:

Решить:

Решение

Обратите внимание, что переменная находится только в левой части уравнения, поэтому это будет сторона переменной, а правая сторона будет стороной константы.Так как левая сторона — переменная сторона, это неуместно. Он вычитается из так для «отмены» вычитания, прибавляется к обеим сторонам.

Решить:

Решить:

Решите уравнение с переменными с обеих сторон

Что делать, если в обеих частях уравнения есть переменные? Мы начнем так же, как и выше, — выберем сторону переменной и сторону константы, а затем воспользуемся свойствами равенства вычитания и сложения, чтобы собрать все переменные с одной стороны и все константы с другой стороны.Помните, что то, что вы делаете с левой частью уравнения, вы должны делать и с правой.

Решить:

Решение

Здесь переменная находится с обеих сторон, но константы появляются только с правой стороны, поэтому давайте сделаем правую часть «постоянной». Тогда левая сторона будет «переменной» стороной.

Решить:

Решить:

Решить:

Решение

Единственная постоянная находится в левой части уравнения, а переменная — в обеих сторонах.Оставим константу слева и соберем переменные справа.

Решить:

Решить:

Решить:

Решение

Единственная константа находится справа, поэтому пусть левая сторона будет стороной переменной.

Решить:

Решить:

Решение уравнений с переменными и константами с обеих сторон

Следующий пример будет первым, в котором переменные и будут постоянными по обе стороны уравнения.Как и раньше, мы соберем члены переменных в одну сторону, а константы — в другую.

Решить:

Решить:

Мы кратко опишем предпринятые шаги, чтобы вы могли легко к ним обратиться.

Решите уравнение с переменными и константами с обеих сторон.

  1. Выберите одну сторону как переменную, тогда другая будет постоянной стороной.
  2. Соберите переменные члены в сторону переменных, используя свойство равенства сложения или вычитания.
  3. Соберите константы с другой стороны, используя свойство равенства сложения или вычитания.
  4. Сделайте коэффициент переменной, используя свойство равенства умножения или деления.
  5. Проверьте решение, подставив его в исходное уравнение.

Хорошей идеей будет сделать сторону переменной той, в которой переменная имеет больший коэффициент. Обычно это упрощает арифметику.

Решить:

Решение

У нас слева и справа.Так как левую часть сделайте «переменной» стороной.

Решить:

Решить:

Решить:

Решение

Это уравнение имеет слева и справа. Так как правую сторону сделайте переменной стороной, а левую — постоянной стороной.

Обратите внимание, что мы могли бы сделать левую часть переменной стороной вместо правой, но это привело бы к отрицательному коэффициенту при переменной составляющей. Хотя мы можем работать с негативом, вероятность ошибки при работе с позитивом меньше.Описанная выше стратегия помогает избежать негатива!

Решить:

Решить:

Чтобы решить уравнение с дробями, мы по-прежнему выполняем те же шаги, чтобы получить решение.

Решить:

Решение

Т.к. левую сторону сделайте переменной стороной, а правую — постоянной стороной.

Решить:

Решить:

Мы проделываем те же шаги, когда в уравнении есть десятичные дроби.

Решить:

Решение

Т.к. левую сторону сделайте переменной стороной, а правую — постоянной стороной.

Решить:

Решить:

Решение уравнений с использованием общей стратегии

Каждый из первых нескольких разделов этой главы имел дело с решением одной конкретной формы линейного уравнения. Пришло время разработать общую стратегию, которую можно использовать для решения любого линейного уравнения . Мы называем это общей стратегией .Для решения некоторых уравнений не потребуется выполнять все шаги, но для многих потребуется. Если сначала максимально упростить каждую часть уравнения, остальные шаги будут проще.

Используйте общую стратегию для решения линейных уравнений.

  1. Максимально упростите каждую часть уравнения. Используйте свойство Distributive, чтобы удалить скобки. Комбинируйте похожие термины.
  2. Соберите все переменные члены в одну сторону уравнения. Используйте свойство равенства сложения или вычитания.
  3. Соберите все постоянные члены с другой стороны уравнения. Используйте свойство равенства сложения или вычитания.
  4. Сделайте коэффициент при переменной составляющей равным Использовать свойство равенства умножения или деления. Сформулируйте решение уравнения.
  5. Проверьте решение. Подставьте решение в исходное уравнение, чтобы убедиться, что результат верный.

Решить:

Решить:

Решить:

Решить:

Решить:

Решить:

Решить:

Решить:

Решить:

Решить:

Решение

Будьте осторожны при распространении негатива.

Решить:

Решить:

Решить:

Решить:

Решить:

Решить:

Решить:

Решить:

Во многих приложениях нам придется решать уравнения с десятичными знаками. Та же общая стратегия будет работать для этих уравнений.

Решить:

Решить:

Решить:

Ключевые понятия

  • Решите уравнение с переменными и константами с обеих сторон
    1. Выберите одну сторону как переменную, тогда другая будет постоянной стороной.
    2. Соберите переменные члены в сторону переменных, используя свойство равенства сложения или вычитания.
    3. Соберите константы с другой стороны, используя свойство равенства сложения или вычитания.
    4. Сделайте коэффициент переменной равным 1, используя свойство равенства умножения или деления.
    5. Проверьте решение, подставив его в исходное уравнение.
  • Общая стратегия решения линейных уравнений
    1. Максимально упростите каждую часть уравнения.Используйте свойство Distributive, чтобы удалить скобки. Комбинируйте похожие термины.
    2. Соберите все переменные члены в одну сторону уравнения. Используйте свойство равенства сложения или вычитания.
    3. Соберите все постоянные члены с другой стороны уравнения. Используйте свойство равенства сложения или вычитания.
    4. Сделайте коэффициент при переменной составляющей равным 1. Используйте свойство равенства умножения или деления. Сформулируйте решение уравнения.
    5. Проверьте решение. Подставьте решение в исходное уравнение, чтобы убедиться, что результат верный.
Практика ведет к совершенству

Решите уравнение с константами с обеих сторон

В следующих упражнениях решите уравнение для переменной.

Решите уравнение с переменными с обеих сторон

В следующих упражнениях решите уравнение для переменной.

Решите уравнение с переменными и константами с обеих сторон

В следующих упражнениях решите уравнения для переменной.

Решение уравнения с использованием общей стратегии

В следующих упражнениях решите линейное уравнение, используя общую стратегию.

Письменные упражнения

Почему при решении уравнения с переменными с обеих сторон обычно лучше выбирать сторону с большим коэффициентом в качестве стороны переменной?

Решите уравнение, объясняющее все этапы вашего решения.

Какой первый шаг вы делаете при решении уравнения Объясните, почему это ваш первый шаг.

Решите уравнение, объясняющее все этапы вашего решения, как в примерах в этом разделе.

Своими словами перечислите шаги в Общей стратегии решения линейных уравнений.

Объясните, почему вам следует максимально упростить обе стороны уравнения, прежде чем собирать переменные члены в одну сторону и постоянные члены — в другую.

Самопроверка

ⓐ После выполнения упражнений используйте этот контрольный список, чтобы оценить свое мастерство в достижении целей этого раздела.

ⓑ Что этот контрольный список говорит вам о вашем мастерстве в этом разделе? Какие шаги вы предпримете для улучшения?

Тем по алгебре: Решение уравнений

Урок 8: Решение уравнений

/ ru / algebra-themes / упрощающие-выражения / content /

Решение уравнений

В предыдущем разделе мы говорили о упрощающих выражениях .В этом разделе мы поговорим о решениях уравнений. Уравнения — это два выражения, равных друг другу с помощью знака равенства (=). Когда мы упрощаем выражения, наша конечная цель состоит в том, чтобы не осталось никаких операций.

Когда мы решаем уравнения, наша конечная цель — выяснить, чему равна переменная (или буква), поместив переменную отдельно по одну сторону от знака равенства и само число с другой. Мы собираемся достичь этой цели, выполнив два важных шага:

  1. Упростите каждое выражение по обе стороны от знака равенства.
  2. Используйте обратные операции для отмены.

Звучит сложно? Мы разберем его, чтобы было проще. Давайте посмотрим на пример:

5x — 4x — 6 = 18

Мы можем начать решать так же, как начинали бы упрощать выражение, проверяя порядок операций. Мы хотим максимально упростить каждую сторону знака равенства сначала . Глядя на наше уравнение, нет скобок или показателей степени, и нечего умножать или делить, поэтому мы просто начнем складывать и вычитать.Первая часть проста: 5 x — 4 x равно 1 x , или просто x .

Отмена с обратными операциями

Теперь у нас осталось это уравнение:

х — 6 = 18

Мы не можем вычесть 6 из x , потому что они не , как термины (наш урок чтения алгебраических выражений объясняет это более подробно). Но x — 6 = 18 все еще недостаточно упрощен. В конце концов, мы ищем значение x , а не значение x — 6.

Чтобы решить это уравнение, нам нужно получить x только на одной стороне знака равенства. Чтобы переместить -6 на другую сторону от знака равенства, мы можем использовать , обратный — или противоположный — -6. Это будет 6. Другими словами, мы можем прибавить шесть к обеим сторонам уравнения.

В левой части уравнения -6 плюс 6 равно 0, а x -0 равно x . Справа 18 плюс 6 равно 24, поэтому x = 24.Теперь наше уравнение упрощено. Мы упростили его, используя , инверсный того, от чего мы хотели избавиться.

Это также называется , отменяющим , потому что оно позволяет вам отменить или избавиться от части уравнения. Это не значит, что вы можете просто вычеркнуть любую часть уравнения, которую не хотите решать (хотя это значительно упростит алгебру!). Вы должны соблюдать несколько правил.

Во-первых, вы заметили, что мы добавили 6 к обеим сторонам нашего уравнения? Это потому, что две стороны уравнения всегда должны быть равными — в конце концов, это то, что означает знак равенства.Каждый раз, когда вы делаете что-то дополнительно к одной стороне уравнения, вы должны делать то же самое с другой. Поскольку мы добавили 6 к -6 на левой стороне , нам также пришлось добавить ее к 18 на правой стороне .

Во-вторых, помните, как мы прибавили шесть, где в исходном выражении говорилось, что вычитает ? Мы сделали это, потому что 6 — это противоположность -6. Чтобы отменить часть выражения, вам нужно использовать ее противоположную или инверсную. Противоположностью вычитания является , сложение , и, как вы могли догадаться, противоположность сложения — , вычитание .

Посмотрите видео ниже, чтобы увидеть, как эта проблема решена.

А как насчет умножения и деления? Это тоже противоположности, и вы также можете их отменить. Например, как получить a только в этом уравнении слева от знака равенства?

5a = 30

Поскольку a умножается на , на 5, вы можете разделить обе стороны задачи на 5. 5 a разделить на 5 равно a и 30 разделить на 5 равно 6, поэтому упрощенная версия этого уравнения будет выглядеть так:

а = 6

Посмотрите видео ниже, чтобы увидеть, как эта проблема решена.

Многоступенчатые уравнения

Давайте посмотрим на другой пример:

4 (2x + 3) = 68

Во-первых, нам нужно посмотреть, можно ли что-нибудь упростить. Помните, в предыдущем разделе мы говорили о числе вне скобок, означающем умножение? В соответствии с этим, мы можем умножить 4 · 2x и 4 · 3. 4 · 2x будет 8x и 4 · 3 будет 12 .

8x + 12 = 68

Это дает нам 8x + 12 = 68 .

Теперь, когда обе стороны знака равенства упрощены, нам нужно будет использовать отмену, чтобы получить x отдельно. Прямо сейчас у нас есть две вещи, которые нам нужно переместить, 8 и 12. Мы добавляем 12, поэтому мы должны вычесть, чтобы переместить их. Мы также умножаем x на 8, поэтому мы будем делить, чтобы переместить его. Но какой из них мы двинемся в первую очередь?

Помните, что для отмены используется обратных — или противоположных — операций. Поскольку мы используем противоположные операции для перемещения объектов, мы собираемся использовать напротив порядка операций, чтобы решить, в каком порядке их перемещать.

Порядок операций гласит, что мы упростим умножение и деление перед сложением и вычитанием, поэтому мы собираемся сделать наоборот. Сначала мы будем использовать сложение / вычитание, а затем умножение / деление.

Сначала вычтем 12 с обеих сторон:

Поскольку 12–12 равняется 0, слева остается 8x. Поскольку 68-12 это 56, у нас остается 56 справа.

Наконец, разделим. 56/8 = 7

х = 7

Готово! Это означает, что для 4 (2x + 3) = 68 x должен быть равен 7.

Посмотрите видео ниже, чтобы увидеть, как эта проблема решена.

Практика!

Давайте попрактикуемся в том, что вы только что узнали, решив еще несколько задач. Помните, что для упрощения мы будем использовать порядок операций : и , отменяя .

Обратите внимание на шаги, которые мы предпринимаем для упрощения этих выражений — через некоторое время у вас будет возможность решить несколько самостоятельно.

Проблема 1

Упростите это выражение, чтобы найти значение x :

6x + 2 3 = 74

Найдите минутку, чтобы подумать, что бы вы сделали в первую очередь.Возможно, вы даже захотите достать лист бумаги, чтобы увидеть, как вы можете упростить это самостоятельно. Когда будете готовы, продолжайте читать, чтобы узнать, как мы получили правильный ответ.

Как и на предыдущей странице, мы начнем с того, что посмотрим, можем ли мы что-нибудь сделать с порядком операций . Это выражение имеет две операции: сложение и показатель степени .

6x + 2 3 = 74

Согласно порядку операций, нам нужно сначала вычислить показатель степени.Это 2 3 , что равно 2 ⋅ 2 ⋅ 2 , или 8.

6x + 2 3 = 74

Порядок операций гласит, что мы должны добавить следующее, но мы не можем добавить 6 x + 8 — переменная с коэффициентом 6 x может быть добавлена ​​только к другому подобному члену. (Другими словами, число с переменной x может быть добавлено только к другому числу с переменной x .) Чтобы получить 6 x самостоятельно, нам нужно отменить + 8.

6x + 8 = 74

Мы можем сделать это с напротив из 8, что равно — 8. Мы вычтем 8 с обеих сторон от знака равенства. 8-8 равно 0. 74-8 равно 66.

Мы почти закончили. Все, что осталось сделать, это избавиться от 6 из 6 x . Помните, что 6 x — это просто еще один способ записи 6 ⋅ x .

6x = 66

Поскольку 6 и x умножаются на на , мы можем отменить 6, сделав обратное: делим .

6 x /6 равно x и 66/6 равно 11, поэтому x = 11. Готово!

х = 11

Как вы могли заметить, вам не нужно соблюдать порядок операций после того, как вы начали отмену. Все, что имеет значение, — это , при котором обе стороны выражения равны . Фактически, лучше всего отменить сложение и вычитание перед .

Проблема 2

Попробуем другую задачу.Упростим и .

4 (3 года — 8) = 4

Эта задача немного отличается от предыдущей, но использует те же навыки. Вот как это решить:

В соответствии с порядком операций нам сначала нужно упростить выражение в скобках . Однако мы не можем вычесть 8 из 3 y — мы не можем вычесть число из переменной.

4 (3 года — 8) = 4

Поскольку 4 стоит рядом с круглыми скобками, мы должны умножить , указанное в скобках, на 4.(Запутались? Просмотрите наш урок по чтению алгебраических выражений).

4 (3y -8) = 4

4 3 y равно 12 y и 4 ⋅ -8 равно -32. Вы также не можете вычесть 32 из 12 y , поэтому для дальнейшего упрощения этого выражения нам придется начать отменять.

12лет — 32 = 4

Давайте сначала избавимся от -32. Противоположность -32 — 32, поэтому мы прибавим к обеим сторонам 32. — 32 + 32 равно 0, а 4 + 32 равно 36.

Мы почти закончили. Нам просто нужно отменить из 12 в 12 y . Помните, что 12 y также можно записать как 12 ⋅ y .

12 y = 36

Поскольку 12 и y умножаются на на , мы можем сократить 12 на , разделив .

12 y /12 равно y , а 36/12 равно 3. Мы сделали это: y равно 3.

y = 3

Ваша очередь

Попробуйте решить следующие несколько проблем самостоятельно.Ответы ниже.

Проблема 1

Упростите это выражение, чтобы найти значение x :

-2 + x / 5-3 = 0

Проблема 2

Найдите значение y :

3 (y + 2y) = 36

Проблема 3

Найдите значение r :

300р — 60р + 10 2 = -380

Ответы:
  1. x = 25
  2. y = 4
  3. r = -2

Более длинные уравнения

Хотите верьте, хотите нет, но теперь у вас есть инструменты для упрощения многих выражений, даже таких сложных на вид, как это:

3x — 24 ⋅ 2 = 8x + 2

Это может показаться более сложным, чем задачи, которые вы решили на предыдущей странице, но вы будете использовать те же навыки, чтобы решить эту.Основное различие между этим выражением и другими, которые вы решили, состоит в том, что у этого есть переменная и по крайней мере одно число на по обе стороны от знака равенства , так что вам придется немного больше компенсировать.

Вам также нужно будет выбрать, хотите ли вы, чтобы переменная была слева или справа от знака равенства в вашем упрощенном выражении. На самом деле это не имеет значения — ответ будет одинаковым в любом случае, но в зависимости от задачи вы можете обнаружить, что математика кажется проще, чем в другой.Тем не менее, несмотря ни на что, ваше упрощенное уравнение должно иметь только переменную с одной стороны уравнения и только число с другой.

Давайте попробуем решить задачу вверху страницы: 3 x — 24 ⋅ 2 = 8 x + 2.

Во-первых, мы захотим разобраться в том, что мы можем, с порядком операций. Похоже, все, что мы можем сделать, это умножить -24 ⋅ 2. Все остальное требует сложения или вычитания в отличие от терминов: — 24 ⋅ 2 равно -48.

3x -24 ⋅ 2 = 8x + 2

Давайте попробуем получить x на левой стороне знака равенства и число справа .Начнем с исключения -48 слева. Мы можем сделать это, добавив 48 к обеим сторонам. -48 + 48 равно 0, а 2 + 48 равно 50.

Поскольку мы решили, что x будет на левой стороне , мы должны избавиться от 8 x справа. Мы можем сделать это, вычтя 8 x с обеих сторон. 8 x — 8 x равно 0, а 3 x — 8 x равно -5 x .

Теперь все, что осталось сделать, это избавиться от -5 в -5 x . Поскольку -5 x — это способ записи -5 ⋅ x , мы можем отменить его, разделив обе стороны на -5. -5 x / -5 равно x , а 50 / -5 равно 10.

Готово! x равно -10.

х = -10

Как видите, упрощение этого уравнения на самом деле было не намного сложнее, чем упрощение любого из других уравнений в этом уроке — просто это заняло немного больше времени.

Посмотрите видео ниже, чтобы увидеть, как эта проблема решена.

Практика!

Теперь ваша очередь. Попробуйте упростить эти длинные выражения.

Проблема 1

Решите относительно и .

-46 -2i = 42 + 7i ⋅ 6

Проблема 2

Решить относительно j .

90j / 5 + 2 2 = 140 + j

Проблема 3

Решить относительно k . (Подсказка: ваш окончательный ответ будет дробным.)

3 + (3k + 6k) = 3k + 5

ответы
  1. i = -2
  2. j = 8
  3. k = 1/3

Уравнения с более чем одной переменной

Иногда вы можете увидеть уравнение с более чем одной переменной, например, это:

2x + 6y -10 = 38

Если выражение содержит более одной переменной, вы не сможете упростить его полностью — недостаточно информации. Вместо этого в задачах с уравнениями с несколькими переменными обычно предлагается решить для одну переменных.Вы максимально упростите его, добавив переменную, которую вы решаете, с одной стороны уравнения, а любые другие числа и переменные — с другой. Упростим приведенное выше выражение: 2 x +6 y — 10 = 38.

Мы ничего не можем сделать с порядком операций, так что давайте начнем отменять. Нам нужно только x на левой стороне , поэтому мы постараемся расположить все остальное справа.

2х + 6лет — 10 = 38

Сначала отменим -10.Противоположность -10 равна 10, поэтому мы добавим по 10 к обеим сторонам. -10 + 10 равно 0, а 38 + 10 равно 48.

Далее избавляемся от 6 y . Мы вычтем с обеих сторон по . 6 y — 6 y равно 0. Поскольку с другой стороны нечего вычитать, мы просто напишем -6 y справа. (Вы запутались? Это похоже на то, что мы вычли 6 y из ничего , или 0 — и 0-6 y равно -6 y .)

Теперь нам нужно избавиться от 2 в 2 x . Поскольку 2 x — это еще один способ сказать 2 ⋅ x, мы разделим обе стороны на 2, чтобы получить только x слева. 2 x /2 составляет x , а (48-6 y ) / 2 составляет 24-3 y .

Это все, что нужно! Выражение не полностью упрощено — мы все еще не знаем числовое значение x и y — но оно достаточно упрощено, потому что мы можем сказать, что x равно 24 — 3 y .

х = 24 — 3 года

Помните, ваша цель при решении подобных задач состоит не в том, чтобы полностью упростить выражение, а в том, чтобы найти значение одной из переменных.

Это — это , которое фактически можно решить для двух переменных, если у вас есть более одного уравнения с одинаковыми переменными. Это называется системой уравнений. На самом деле мы используем системы уравнений в нашем уроке по задачам дистанционных слов, но мы не обсуждаем, как они работают в целом.Чтобы узнать больше о системах уравнений, посмотрите это видео от Khan Academy.

Посмотрите видео ниже, чтобы увидеть, как эта проблема решена.

Практика!

Проблема 1

Решить относительно r .

88q + 4r — 3 = 5

Проблема 2

Решить для с . (Подсказка: ваш окончательный ответ будет дробью со знаминателем r .)

(13ср) / 2 = 39

Проблема 3

Решить для м .

6 м. — 30 чел. / 5 = 12

ответы
  1. r = 2-22 q
  2. s = 6/ r
  3. m = 2 + p

Проверка вашей работы

Важно проверять свою работу по алгебре, особенно когда вы только начинаете. К счастью, проверить свою работу, когда вы упрощаете уравнения, довольно просто. Все, что вам нужно сделать, это заменить переменную в уравнении значением, которое вы нашли при его упрощении.Чтобы увидеть, как это работает, давайте вернемся к одному из упрощенных ранее уравнений:

4 (3 года — 8) = 4

Мы обнаружили, что y равно 3. Посмотрим, правильно ли мы получили ответ.

Вот наше исходное уравнение. y — наша переменная, поэтому мы заменим ее найденным значением: 3.

4 (3 года — 8) = 4

Вот как выглядит уравнение с 3 вместо y . Теперь посмотрим, верно ли уравнение.Если левая сторона равна правой, наш ответ правильный.

4 (3 ⋅ 3–8) = 4

Мы будем следовать порядку операций, сначала скобки. 3 ⋅ 3 равно 9, а 9-8 равно 1.

4 (1) = 4

Теперь, когда мы упростили скобки, все, что нам нужно сделать, это умножить 4 на 1.

4 (1) = 4

4 ⋅ 1 равно 4. Обе части нашего уравнения равны, поэтому наш ответ правильный!

4 = 4

Это все, что нужно! Проверять каждое упрощенное выражение — хорошая привычка, и вы обнаружите, что проверка своей работы обычно занимает меньше времени, чем на упрощение уравнения в первую очередь.

Попробуем еще:

Выражение, на которое мы будем смотреть, 5 x + 3 = 23 + x . Мы проверяем правильность решения x = 4.

5x + 3 = 23 + x

Сначала заменим переменную x на 4.

5 ⋅ 4 + 3 = 23 + 4

Чтобы проверить нашу работу, нам нужно упростить обе стороны выражения. Начнем с левой стороны . По порядку действий нам нужно сначала умножить, а потом сложить. 5 ⋅ 4 равно 20, и когда вы прибавите к этому 3 , вы получите 23.

5 ⋅ 4 + 3 = 23 + 4

Теперь нам нужно упростить правую часть: 23 + 4 равно 27.

23 = 23 + 4

Наше уравнение не может быть правильным — 23 и 27 не равны . Теперь мы знаем, что x не равно 4. Другими словами, ответ неправильный .

23 = 27

Как вы только что видели, если вы проверяете проблему, и окончательное выражение — , а не — сбалансированное уравнение, ваш ответ — , а не правильное.Найдите время, чтобы вернуться и снова упростить исходное уравнение. Со второй попытки обратите особое внимание на порядок операций и убедитесь, что вы правильно складываете, вычитаете, умножаете и делите.

Хотите еще раз проверить последнюю проблему? На этот раз проверьте это с помощью x = 5.

Практика!

Проблема 1

Проверьте эту проблему. u = 6 правильный ответ? Если нет, то что?

ед (3 + 8) / 2 = 33

Проблема 2

Проверьте эту проблему.Правильный ли ответ против = 5? Если нет, то что?

В / 5 + 20 В = 19 В + 12

Проблема 3

Проверьте эту проблему. w = 8 правильный ответ? Если нет, то что?

5 Вт + 3 = 4 Вт + 10

ответы
  1. Да, ответ правильный.
  2. №; v = 10.
  3. Нет; w = 7.

/ ru / algebra-themes / Introduction-to-word-tasks / content /

Калькулятор дробей

Калькулятор выполняет базовые и расширенные операции с дробями, выражениями с дробями, объединенными с целыми числами, десятичными знаками и смешанными числами.Он также показывает подробную пошаговую информацию о процедуре расчета дроби. Решайте задачи с двумя, тремя или более дробями и числами в одном выражении.

Правила для выражений с дробями:

Дроби — используйте косую черту «/» между числителем и знаменателем, т.е. для пяти сотых введите 5/100 . Если вы используете смешанные числа, не забудьте оставить один пробел между целой и дробной частью.
Косая черта разделяет числитель (число над дробной чертой) и знаменатель (число ниже).

Смешанные числа (смешанные дроби или смешанные числа) записываются как ненулевое целое число, разделенное одним пробелом и дробью, то есть 1 2/3 (с тем же знаком). Пример отрицательной смешанной дроби: -5 1/2 .
Поскольку косая черта является одновременно знаком для дробной линии и деления, мы рекомендуем использовать двоеточие (:) в качестве оператора деления дробей, то есть 1/2: 3 .

Десятичные числа (десятичные числа) вводятся с десятичной точкой . , и они автоматически конвертируются в дроби — i.е. 1,45 .

Двоеточие : и косая черта / являются символом деления. 1/2
• сложение дробей и смешанных чисел: 8/5 + 6 2/7
• деление целого и дробного числа: 5 ÷ 1/2
• комплексные дроби: 5/8: 2 2/3
• десятичное в дробное: 0.625
• Дробь в десятичную: 1/4
• Дробь в процент: 1/8%
• сравнение дробей: 1/4 2/3
• умножение дроби на целое число: 6 * 3/4 ​​
• квадратный корень дроби: sqrt (1/16)
• уменьшение или упрощение дроби (упрощение) — деление числителя и знаменателя дроби на одно и то же ненулевое число — эквивалентная дробь: 4/22
• выражение в скобках: 1 / 3 * (1/2 — 3 3/8)
• сложная дробь: 3/4 от 5/7
• кратная дробь: 2/3 от 3/5
• разделите, чтобы найти частное: 3/5 ÷ 2 / 3

Калькулятор следует известным правилам порядка операций .Наиболее распространенные мнемоники для запоминания этого порядка операций:
PEMDAS — Круглые скобки, экспоненты, умножение, деление, сложение, вычитание.
BEDMAS — Скобки, экспоненты, деление, умножение, сложение, вычитание
BODMAS — Скобки, порядок или порядок, деление, умножение, сложение, вычитание.
GEMDAS — Группировка символов — скобки () {}, экспоненты, умножение, деление, сложение, вычитание.
Будьте осторожны, всегда делайте умножение и деление перед сложением и вычитанием .Некоторые операторы (+ и -) и (* и /) имеют одинаковый приоритет и должны вычисляться слева направо.

Задачи на дроби:

следующие математические задачи »

Математическое выражение: решение линейного уравнения

00: 00: 00.210
Этот урок покажет вам основы решения линейного уравнения.

00: 00: 07.100
Рассмотрим уравнение x + 5 = 8. Чтобы решить это уравнение, наша цель — найти значение x.Это означает, что чему равен x?

00: 00: 19.230
Для этого, по логике вещей, мы должны удалить +5.

00: 00: 25.060
Итак, мы добавляем — 5 в левую часть уравнения.

00: 00: 30.140
Отсюда, поскольку мы добавляем -5 к левой части. Вы также должны добавить -5 с правой стороны.

00: 00: 38.240
Почему мы должны это делать? Это потому, что здесь стоит знак равенства. Итак, любые термины, которые вы добавляете в левую часть, должны быть одинаково сбалансированы правой стороной.

00: 00: 50.240
Теперь +5 и -5 отменяют друг друга. Остается x = 8-5.

00: 00: 59.210
Теперь мы можем решить это уравнение, используя минус 8 и 5. Это дает 3. Ответ: x = 3.

00: 01: 10.060
Теперь, поняв логику, лежащую в основе, мы можем используйте ярлык для решения этого уравнения.

00: 01: 18.010
Мы можем просто переместить +5 на другую сторону. Помните, когда вы его перемещаете, просто нужно сменить знак на противоположный знак.В этом случае меняем знак с положительного на отрицательный.

00: 01: 32.010
Позвольте показать вам. Видите, положительный знак меняется на отрицательный, когда я перемещаю его. Итак, зачем нам менять знак?

00: 01: 43.030
Это потому, что когда мы перемещаем +5, это то же самое, что и удаление +5 путем добавления с -5.

00: 01: 51.070
Итак, когда вы кладете его на другую сторону, оно должно стать -5. Заметьте, что это работает точно так же, если я верну -5 назад.

00:02:05.220
Теперь минус 8 с 5. Это дает 3.

00: 02: 13.220
Затем давайте решим x -2 = 8. Помня о цели, нам нужно удалить -2, чтобы удалить -2, мы добавить +2 к левой части

00: 02: 29.110
Поскольку мы добавляем +2 к левой части, нам нужно уравновесить, добавив +2 к правой части.

00: 02: 37.230
Теперь, -2 и +2 компенсируют друг друга. Мы решаем это уравнение, складывая 8 с 2. Это дает 10.

00: 02: 47.110
Теперь у нас x = 10.

00: 02: 51.090
Мы можем использовать ярлык, чтобы найти x. Для этого мы можем просто переместить -2 на другую сторону.

00: 03: 00.160
Опять надо поменять знак. Итак, отрицательный знак становится положительным знаком. Если я верну его обратно, станет -2.

00: 03: 16.060
Опять же, зачем нам менять знак? Это потому, что, когда мы перемещаем -2, мы фактически удаляем -2, добавляя его с +2.

00: 03: 28.160
Итак, когда вы кладете его на другую сторону, он становится +2.Это та же идея, что и в предыдущем примере.

00: 03: 37.230
Теперь добавьте 8 с +2. Мы получаем 10.

00: 03: 43.230
Следующий пример решения линейного уравнения: решите 3x = 9. Помня о цели, мы знаем, что нам нужно удалить 3.

00: 03: 54.050
Для этого логически мы делим 3x на 3. Аналогично, чтобы уравнение оставалось сбалансированным, мы также должны разделить 9 на 3.

00: 04: 06.170
Теперь, 3 делится на 3, взаимно компенсируя друг друга. Остается x = 9/3.9 делится на 3 дает 3.

00: 04: 19.140
Итак, ответ — x = 3. Давайте воспользуемся ярлыком для решения этого уравнения.

00: 04: 28.030
Поскольку 3 фактически умножается на x, когда мы перемещаем 3 в другую сторону, оно становится делением. Обратите внимание, что знак не изменился. Позвольте мне объяснить почему.

00: 04: 41.020
Это потому, что при делении на обе стороны на 3. манипуляции со знаком отсутствуют. Следовательно, нам не нужно менять знак.

00: 04: 51.200
Чтобы решить это уравнение, разделим 9 на 3.Итак, ответ x равен 3.

00: 05: 02.000
Следующий пример решения линейного уравнения. Решите x / 7 = 2. Чтобы найти x, нам нужно умножить левую часть на 7. Чтобы сбалансировать, мы также должны умножить правую часть на 7.

00: 05: 16.240
Теперь 7 и 7 отменяют друг с другом. Отсюда получаем x = 7 скобку два. 7 умножить на 2 дает 14.

00: 05: 29.210
Что касается ярлыка. Поскольку 7 используется для деления x, когда мы поднимаем его, его операция становится умножением. Обратите внимание, что вы должны заключить в скобки все термины на этой стороне.

00: 05: 44.100
Обратите внимание, что знак не изменился. Причина этого аналогична предыдущему примеру.

00: 05: 51.080
Теперь умножьте 7 x 2. Вы получите x = 14.

00: 05: 56.210
На этом урок по решению линейного уравнения закончился.

Добавить комментарий

Ваш адрес email не будет опубликован. Обязательные поля помечены *